Invariantes. Todo lo que debías saber para la OMA

78
Matemática e Invariantes 1

Transcript of Invariantes. Todo lo que debías saber para la OMA

Page 1: Invariantes. Todo lo que debías saber para la OMA

Matemática e Invariantes

1

Page 2: Invariantes. Todo lo que debías saber para la OMA

2

Page 3: Invariantes. Todo lo que debías saber para la OMA

Indice1. Pregunta y Advertencia 4

2. El Engaño 52.1. Introducción . . . . . . . . . . . . . . . . . . . . . . . . . . . . . . . . 62.2. Primeros pasos . . . . . . . . . . . . . . . . . . . . . . . . . . . . . . 82.3. Luces . . . . . . . . . . . . . . . . . . . . . . . . . . . . . . . . . . . . 142.4. Piedras . . . . . . . . . . . . . . . . . . . . . . . . . . . . . . . . . . . 182.5. Fichas . . . . . . . . . . . . . . . . . . . . . . . . . . . . . . . . . . . 202.6. Un poco de Geometría Proyectiva . . . . . . . . . . . . . . . . . . . . 242.7. Cartas . . . . . . . . . . . . . . . . . . . . . . . . . . . . . . . . . . . 312.8. Rompecabezas . . . . . . . . . . . . . . . . . . . . . . . . . . . . . . . 362.9. Pizarrones, Tablas y Grupos . . . . . . . . . . . . . . . . . . . . . . . 392.10. Senku . . . . . . . . . . . . . . . . . . . . . . . . . . . . . . . . . . . 432.11. Ranas . . . . . . . . . . . . . . . . . . . . . . . . . . . . . . . . . . . 462.12. Monovariantes . . . . . . . . . . . . . . . . . . . . . . . . . . . . . . . 482.13. Un problema y 2 soluciones . . . . . . . . . . . . . . . . . . . . . . . 512.14. Diamante Azteca . . . . . . . . . . . . . . . . . . . . . . . . . . . . . 532.15. Pulgas . . . . . . . . . . . . . . . . . . . . . . . . . . . . . . . . . . . 59

3. Apéndices 613.1. Sumas infinitas . . . . . . . . . . . . . . . . . . . . . . . . . . . . . . 623.2. Sumas de potencias . . . . . . . . . . . . . . . . . . . . . . . . . . . . 653.3. Sucesiones recurrentes . . . . . . . . . . . . . . . . . . . . . . . . . . 673.4. Permutaciones . . . . . . . . . . . . . . . . . . . . . . . . . . . . . . . 693.5. Menelao . . . . . . . . . . . . . . . . . . . . . . . . . . . . . . . . . . 71

4. Problemas 754.1. Algunos problemas para pensar . . . . . . . . . . . . . . . . . . . . . 76

3

Page 4: Invariantes. Todo lo que debías saber para la OMA

1. Pregunta y Advertencia

Advertencia: Cada solución que escribía despertaba el mismo miedo, que ibana leer la solución antes de pensar el problema y así no sólo perderían la oportunidad depensar el problema sino que entenderían menos la solución. Por eso esta advertencia,es mejor pensar un problema por algún tiempo y que no salga que leer su solución en5 minutos y olvidarse de él. Piensen cada problema, no se apresuren a leer su solución.

Pregunta: “Hacer matemática no es cuestión de acumular ideas, trucos o teo-remas para que cada vez que se nos presente un problema repasemos la lista hastadar con el teorema indicado.....ni siquiera es importante conocer toda clase de teo-remas, trucos o técnicas....“ ¿Por qué entonces escribir algo que parece un manualpara resolver problemas usando invariantes? ¿No parece esto tratar de listar todos losinvariantes habidos y por haber?

4

Page 5: Invariantes. Todo lo que debías saber para la OMA

2. El Engaño

5

Page 6: Invariantes. Todo lo que debías saber para la OMA

2.1. Introducción

Supongamos que tenemos un objeto que va cambiando de acuerdo a ciertas reglas.Nos podemos preguntar:

¿En qué puede transformarse?

¿Cuánto tardará en llegar a cierto estado?

¿Puede cambiar indefinidamente?

¿Puede transformarse en algo en particular?

Los invariantes nos ayudan a responder estas preguntas, la idea es mirar lo queno cambia a medida que nuestro objeto va tranformándose. Veamos un ejemplo, elproblema es de la XI Olimpíada Nacional Argentina.

Se tienen 3 hormigas en los vértices de un cuadrado. En cada turno, unahormiga se puede mover en dirección paralela a la recta que determinanlas otras 2. ¿Es posible que después de algunos turnos las hormigas ocupen3 puntos medios de los lados del cuadrado?

Figura 1: La hormiga en C puede ir hasta D.

6

Page 7: Invariantes. Todo lo que debías saber para la OMA

Solución: No, no es posible. Notemos que el área del triángulo que forman las 3hormigas no varía con los turnos, pues cada vértice se mueve en forma paralela al ladoopuesto1. Si el cuadrado inicial tenía área 1, entonces 3 vértices forman un triángulode área 1

2y 3 puntos medios forman un triángulo con area 1

4. Entonces no es posible

que las hormigas terminen en 3 puntos medios de los lados del cuadrado. ‡

Figura 2: El área nunca cambia

En este caso el objeto era la posición de las 3 hormigas y el “Invariante“ el áreadel triángulo que formaban.

1Lo que estamos usando es que si tenemos 2 triángulos ABC y ABD con AB//CD entoncesárea(ABC) = área(ABD)

7

Page 8: Invariantes. Todo lo que debías saber para la OMA

2.2. Primeros pasos

Para empezar, algunos ejemplos sencillos. En todos los problemas tenemos algoque va cambiando de acuerdo a ciertas reglas, lo que tenemos que hacer es mirar concuidado y encontrar algo que no cambia.

• En el pizarrón hay 10 signos + y 5 signos −. En cada turno se debeelegir dos de ellos, borrarlos y poner un + si eran iguales y un − si erandistintos. Probar que después de 14 turnos quedará un signo − solamente.

Si pensamos a cada signo + como un 1 y a cada signo − como un −1, al borrar 2números los debemos reemplazar por su producto, entonces el producto de todos losnúmeros en el pizarrón no cambia. Como inicialmente el producto era 110(−1)5 = −1,entonces luego de 14 turnos cuando sólo queda un número, éste debe ser −1. ‡

• En el pizarrón hay escritos 2 números, en cada turno se borran los 2números y si éstos eran a y b entonces se reemplazan por 2a− b y 2b− a.Si inicialmente estaban escritos el 1458 y 1460. ¿Es posible que luego devarios turnos los números en el pizarrón sean el 715 y el 1024?

Como 2a − b + 2b − a = a + b entonces la suma de los 2 números escritos en elpizarrón nunca cambia. Al principio ésta es 1458 + 1460 = 2918, como 715 y 1024 nosuman 2918 entonces los números en el pizarrón nunca serán 715 y 1024. ‡

• Se tiene un tablero de 8× 8 de ajedrez pintado de blanco y negro de laforma usual. En cada turno se pueden intercambiar 2 columnas o 2 filas.¿Es posible lograr que todas las casillas negras estén en la mitad izquierdadel tablero y las casillas blancas en la mitad derecha?

Notemos que la cantidad de casillas negras en cada columna nunca varía, comoinicialmente había 4 casillas negras en cada columna entonces no se pueden ponertodas las negras en la mitad izquierda ya que tendríamos 8 negras en cada una de lasprimeras 4 columnas. ‡

8

Page 9: Invariantes. Todo lo que debías saber para la OMA

• Inicialmente se tiene la terna (3, 4, 12). En cada paso se puede tomar losnúmeros a y b y cambiarlos por 0,6a− 0,8b y 0,8a + 0,6b. ¿Se puede llegara (4, 6, 12)?

Notemos que:

(0,6a−0,8b)2+(0,8a+0,6b)2 = 0,36a2−0,48ab+0,64b2+0,64a2+0,48ab+0,36b2 = a2+b2

Entonces la suma de los cuadrados de los 3 números nunca cambia. Es decir que sillegamos a la terna (a, b, c), luego a2+b2+c2 = 32+42+122 = 169, pero 42+62+122 =196 > 169. Entonces no se puede llegar a (4, 6, 12). ‡

A veces no es tan sencillo encontrar “la cuenta correcta“....

• Un círculo está dividido en 6 sectores (como en la figura), los números1, 0, 1, 0, 0, 0 están escritos en ellos. En cada turno se puede sumarle 1 ados sectores adyacentes. ¿Es posible lograr que todos los sectores tenganel mismo número?

Si a1, a2, a3, a4, a5, a6 son los números escritos entonces S = a1−a2+a3−a4+a5−a6

permanece invariante. Inicialmente S = 2, si todos los números se vuelven igualesentonces sería S = 0. ¡Lo que es absurdo! De donde nunca los 6 sectores tendrán elmismo número. ‡

9

Page 10: Invariantes. Todo lo que debías saber para la OMA

• Si se tiene el par {x, y} se lo puede cambiar por el par {y, x} , {x− y, y}o {x + y, y}. ¿Se puede obtener el par {819, 357} a partir del par {1, 2}?¿Qué pares se puede obtener a partir del {1, 2}?

Notemos que el máximo común divisor de los 2 números que forman el par novaría, pues (x, y) = (y, x) = (x− y, y) = (x + y, y). Como (1, 2) = 1 y (819, 357) = 21entonces no es posible llegar de {1, 2} a {819, 357}.

Si desde {1, 2} se puede llegar a {a, b} entonces (a, b) = 1. Para ver que desde{1, 2} se puede llegar a cualquier par de números coprimos notemos que si de unpar se puede llegar a otro, entonces desde el último se puede volver al primero (estose debe a que las operaciones son reversibles). Entonces tenemos que ver que desde{a, b} se puede llegar a {1, 2}. Procedemos de la siguiente forma, dado {a, b} siemprehagamos que el primer número sea más grande que el segundo y luego le restamos elsegundo al primero. Repetimos lo anterior hasta que obtengamos un 0. Para ver queefectivamente esto debe ocurrir supongamos lo contrario y notemos que los númerossiempre se mantienen no negativos pero en cada turno se van haciendo más y máschicos lo que es absurdo. Si llegamos a {0, m} entonces m = (0, m) = (a, b) = 1 dedonde m = 1. Es fácil ver que desde {0, 1} se puede llegar a {1, 2}. ‡

Es muy común que si bien la suma o el producto no permanecen invariantes, sí lohace su paridad o su resto en la división por 3 (o por algún otro número elegido deforma adecuada).

• En una isla hay 15 camaleones azules, 17 rojos y 19 verdes. Cuando 2camaleones de distinto color se encuentran ambos cambian al tercer color.¿Es posible que en algún momento todos los camaleones se vuelvan delmismo color?

Pongámosle a cada camaleón azul un sombrero con un 1, a cada camaleón rojouno con un 2 y a cada camaleón verde uno con un 3. Cuando un camaleón cambia decolor, también cambia su número (al correspondiente según el nuevo color). Cada vezque se encuentran 2 camaleones y cambian de color, la suma de los números aumentao disminuye en 3 o permanece igual. Entonces si bien no permanece invariante, sí lohace su resto en la división por 3. Inicialmente la suma es 15 + 2 × 17 + 3 × 19 =15 + 34 + 57 = 106 que tiene resto 1 en la división por 3. Si todos los camaleones sevolvieran del mismo color, digamos al correspondiente al número k, entonces la sumasería 51k que es múltiplo de 3 (hay 51 camaleones en total). ¡Absurdo! Entonces loscamaleones no pueden volverse todos del mismo color. ‡

10

Page 11: Invariantes. Todo lo que debías saber para la OMA

• En un pizarrón hay cierta cantidad de letras a, b, c. En cada turno sepuede reemplazar una a y una b por una b, una a y una c por una c,una b y una c por una a, dos a por una a, dos b por una c o dos c poruna b. El objetivo es dejar una sola letra. Probar que sin importar en quéorden realicemos las operaciones, la letra que quede al final siempre serála misma.

Reemplacemos a cada a por un 0, a cada b por un 1 y a cada c por un 2. Notemosque las reglas para borrar letras y escribir otra se pueden reemplazar por la siguientemás simple, se eligen 2 números y se los reemplaza por el resto de su suma en ladivisión por 3. Es fácil ver que la suma de todos los números escritos tendrá siempreel mismo resto en la división por 3 y éste será el número que quede al final. Entoncessin importar el orden en que se realizaron las operaciones, la última letra (o número)será siempre la misma. ‡

• Nos es dado un tablero de 2 × 2 con números en sus casillas. En cadapaso podemos realizar una de las siguientes operaciones:

-Sumar a una fila un múltiplo de la otra.

-Sumar a una columna un múltiplo de la otra.

Si inicialmente había dos 1 en una diagonal y 0 en las otras 2 casillas. ¿Esposible conseguir una fila de 0?

a bc d

1 00 1

→ ∗ ∗0 0

Para cada tablero con los números a, b, c, d en sus casillas como muestra la figura,vamos a llamar a ad − bc su “determinante“. Es fácil ver que éste no varía con lasoperaciones permitidas, pues si por ejemplo sumamos a la segunda fila k veces laprimera entonces

a(d + kb)− b(c + ka) = ad + akb− bc− bka = ad− bc

Inicialmente el determinante es 1 y si conseguimos una fila de ceros sería 0. ¡Absurdo!Entonces no es posible conseguir una fila de ceros. ‡

11

Page 12: Invariantes. Todo lo que debías saber para la OMA

Algunos problemas para pensar

1. Sea n un entero positivo impar y supongamos que en el pizarrón están escritoslos números 1, 2, .., 2n. En cada turno se deben elegir 2 de ellos, digamos a y b,se los borra y se escribe |a − b|. Probar que luego de 2n − 1 pasos, el númeroescrito será impar.

2. Un dragón tiene 100 cabezas. Un caballero le puede cortar 15, 17, 20 o 5 cabezaspero al hacerlo, al dragón le crecen 24, 2, 14 y 17 cabezas nuevas respectiva-mente. ¿Puede el caballero dejar al dragón sin cabezas?

3. Tenemos 50 peones en un tablero de 10 × 10 de forma que 25 de ellos estánen la esquina inferior izquierda de 5× 5 del tablero y los 25 restantes están enla esquina superior derecha de 5× 5 (uno por casilla). En cada turno podemoselegir un peón y hacerlo saltar por sobre cualquier otro peón vecino a la casillasiguiente si ésta se encuentra vacía (cada peón a lo sumo tiene 8 vecinos). ¿Esposible que luego de varios turnos los 50 peones estén en la mitad izquierda deltablero?

4. Borramos el primer dígito de 71996 y se lo sumamos al número obtenido. Se repiteesta operación hasta obtener un número de 10 dígitos. Probar que el número de10 dígitos obtenido tendrá 2 dígitos iguales.

5. En el pizarrón están escritos los números 1, 12, 1

3, .., 1

n. En cada turno se deben

elegir 2 de ellos, digamos a y b, borrarlos y escribir en su lugar ab + a + b. ¿Quénúmero queda después de n− 1 turnos?

6. Sean a1, a2, .., an tales que ai = ±1 y S = a1a2a3a4+a2a3a4a5+...+ana1a2a3 = 0.Probar que n es múltiplo de 4.

7. En cada vértice de un hexágono hay una semilla, en cada turno se puedenescoger 2 de ellas y moverlas 1 vértice en direcciones opuestas. Probar que noes posible llevar todas las semillas a un solo vértice.

8. Nos es dado un tablero de 3 × 3 con números en sus casillas. En cada pasopodemos realizar una de las siguientes operaciones:

-Sumar a una fila un múltiplo de otra.

-Sumar a una columna un múltiplo de otra.

a b cd e fg h i

12

Page 13: Invariantes. Todo lo que debías saber para la OMA

Probar que la siguiente cantidad permanece invariante:

a(ei− fh)− b(di− fg) + c(dh− eg)

Nota: Este número es el determinante de la matriz (tablero) anterior. Es fácilver que si intercambiamos 2 columnas entre sí entonces el determinante siguesiendo el mismo pero con distinto signo. ¿Cuál será el determinante de unamatriz de n× n?

9. Tenemos un tablero de 3 × 3 con números enteros y le podemos aplicar lasmismas operaciones que en el problema anterior. Un menor de 2× 2 del tableroes el determinante del tablero que se obtiene al borrar una fila y una columna.Probar que el máximo común divisor de todos los menores de 2× 2 no varía.

13

Page 14: Invariantes. Todo lo que debías saber para la OMA

2.3. Luces

A veces, si bien el primer invariante que se nos ocurre no funciona, lo único quetenemos que hacer es usar la misma cuenta pero en un conjunto más chico. Veamosalgunos ejemplos.

En cada punto de la figura hay una luz. En cada turno podemos elegir unafila o columna y cambiar el estado de todas las luces en ella. Inicialmentetodas las luces están apagadas salvo la marcada en la figura. ¿Es posiblelograr apagar todas las luces?

No, no es posible. La solución es sencilla, alcanza notar que siempre habrá unacantidad impar de luces prendidas.

Dicho de otra forma, pongamos un 1 por cada luz apagada y un −1 por cadaluz prendida. Cada vez que cambiamos el estado de una luz estamos multiplicandosu número por −1. Como en cada turno cambiamos el estado de 4 luces entoncesmultiplicamos 4 números por −1, de donde el producto de los 16 números permaneceinvariante.

Si logramos apagar todas las luces el producto sería 1, lo que es imposible ya queinicialmente era −1. ‡

¿Qué pasa si en el problema anterior también nos permitimos cambiar elestado de cualquier diagonal de cualquier longitud? (En particular pode-mos cambiar el estado de cualquier esquina). ¿Podemos apagar todas lasluces?

14

Page 15: Invariantes. Todo lo que debías saber para la OMA

El argumento anterior no funciona ya que el producto no permanece invariante,pero..... ¿Por qué multiplicar todos los números? ¿Por qué no buscar un conjunto máspequeño cuyo producto sí permanece invariante?

Si consideramos las 8 luces marcadas de negro en la figura, cada fila, columnao diagonal cambia el estado de 0 o 2 de estas 8 luces, luego el producto de estos 8números debe ser “Invariante“. Como inicialmente es -1 nunca podremos apagar todaslas luces ya que esto supone llegar a un producto = 1. ‡

En cada punto de la figura hay una luz, en cada turno se puede elegirun lado o diagonal del pentágono y cambiar el estado de todas las lucesque están en esa línea. Inicialmente todas las luces están apagadas. ¿Esposible prender las 10 luces?

15

Page 16: Invariantes. Todo lo que debías saber para la OMA

Como antes, reemplacemos las luces apagadas por un 1 y las prendidas por -1. Estavez estamos en la situación opuesta a la anterior. El producto permanece invariantepero el inicial es el mismo al que queremos llegar, lo que no nos lleva a ningún absurdo.

La estrategia es la misma, buscar un conjunto más pequeño tal que el productopermanezca invariante pero que sea distinto el inicial y el final.

Tomemos los 5 puntos exteriores de la figura, los vértices del pentágono. Comocada lado o diagonal toca exactamente 2 de estos 5 puntos entonces en cada turno elproducto de estos 5 números es multiplicado por (−1)2 = 1, es decir que permaneceinvariante.

Inicialmente el producto es 1, pero si conseguimos tener las 10 luces prendidassería −1. ¡Absurdo! Entonces no es posible lograr que todas estén prendidas. ‡

En los ejemplos anteriores vimos 2 situaciones de cómo buscar el invariante ade-cuado cuando el más natural no nos sirve, ya sea porque no permanezca invariante oporque no nos lleve a ningún absurdo. Veamos un ejemplo más.

Se tienen 8 luces en fila. En cada turno se pueden elegir 3 luces consecutivasy cambiar su estado. Si inicialmente todas las luces están apagadas salvolas 2 en los bordes, ¿es posible apagar todas las luces?

Pongamos un 1 por cada luz apagada y un -1 por cada luz prendida. La operaciónpermitida es multiplicar por −1 cualesquiera 3 números seguidos. Notemos que siconsideramos el producto total, éste varía y da lo mismo en la situación inicial y lafinal. Es por ello que debemos buscar un conjunto más pequeño de luces.

Si consideramos las luces marcadas en la figura. Entonces en cada operación vamosa cambiar el estado de 2 luces, que es par. Como inicialmente el producto es −1 nopodemos apagar todas ya que esto nos lleva a un producto = 1. ‡

16

Page 17: Invariantes. Todo lo que debías saber para la OMA

Algunos problemas para pensar

1. Se tienen 16 luces ordenadas en 4 filas y 4 columnas, en cada turno se puedenelegir 3 luces seguidas (en la misma fila o en la misma columna) y cambiar elestado de las 3. Si inicialmente están todas prendidas... ¿Es posible apagar las16 luces?

2. Se tienen 100 luces ordenadas en 10 filas y 10 columnas, en cada turno se puedenelegir 4 luces seguidas (en la misma fila o en la misma columna) y cambiar suestado. Si inicialmente están todas prendidas... ¿Es posible apagar las 100 luces?

3. (Torneo de las Ciudades 2003) En cada casilla de un tablero de 4 × 4 hay unsigno + o un signo −. La operación permitida es elegir una casilla y cambiar elsigno de esa casilla junto con todos los de sus vecinas (que tienen un lado encomún). Determinar cuántos tableros diferentes se pueden obtener.

4. (Torneo de las Ciudades 2004)

a) En cada casilla de un tablero de m × n con m, n ≥ 2 hay un signo + o unsigno −, la operación permitida es cambiar todos los signos de una fila o todoslos signos de una columna. Diremos que un tablero es irreducible si es imposiblemediante operaciones permitidas transformarlo en uno con signos + en todaslas casillas. Demostrar que todo tablero irreducible contiene un cuadrado de2× 2 que es un tablero irreducible de 2× 2.

b) En cada casilla de un tablero de m × n con m, n ≥ 4 hay un signo + oun signo −, la operación permitida es cambiar todos los signos de una fila otodos los signos de una columna o todos los signos de una diagonal. Diremosque un tablero es irreducible si es imposible mediante operaciones permitidastransformarlo en uno con signos + en todas las casillas. Demostrar que todotablero irreducible contiene un cuadrado de 4× 4 que es un tablero irreduciblede 4× 4.

5. Supongamos que tenemos n lámparas y algunos interruptores. Cada interruptorcambia el estado de algunas lámparas. Inicialmente hay algunas lámparas pren-didas. El objetivo es apretar algunos de los interruptores para apagarlas todas.Probar que si esto no es posible entonces se pueden elegir algunas lámparas deforma que no importa qué interruptores apretemos, siempre habrá una cantidadimpar de lámparas prendidas.

17

Page 18: Invariantes. Todo lo que debías saber para la OMA

2.4. Piedras

El problema que sigue fue propuesto por Konstechiv para el II Torneo de lasCiudades en 1981. Nosotros presentamos una pequeña modificación del problemaoriginal.

Consideremos todos los puntos del plano con coordenadas enteras no neg-ativas. Inicialmente tenemos una piedra en (0, 0), una en (0, 1) y otra en(1, 0). En cada paso debemos elegir una piedra en un punto (m, n) y si lospuntos (m,n + 1) y (m + 1, n) se encuentran vacíos, quitamos la piedrade (m, n) y ponemos una en (m,n + 1) y otra en (m + 1, n).

¿Es posible que luego de una cantidad finita de pasos logremos dejar vacíoslos puntos (0, 0), (0, 1) y (1, 0)?

Lo primero que uno piensa es que debería poderse lograr el objetivo. Uno se vetentado a creer que podrá ir llevando cada piedra suficientemente lejos como paraque no moleste la retirada del resto. Luego de experimentar un poco caemos en queno es tan simple. Cada vez tenemos más piedras y éstas cada vez van limitando máslos movimientos del resto. De alguna forma, las piedras más cercanas a (0, 0) sonmás “pesadas“ que las más alejadas ya que les cuesta más moverse. Veamos como losinvariantes nos ayudan a formalizar esto y demostrar que es imposible limpiar los 3puntos (0, 0), (0, 1) y (1, 0).

El peso de una piedra en el punto (m,n) será 12m+n . Nuestro invariante es la suma de

los pesos de todas las piedras, que llamaremos peso total. Notemos que efectivamenteesta cantidad no varía en cada paso pues:

1

2m+n=

1

2m+n+1+

1

2m+1+n

Inicialmente el peso total es 1 + 12

+ 12

= 2. Como en cada punto a lo sumopuede haber una piedra entonces es imposible llegar a una situación con los puntos(0, 0), (0, 1), (1, 0) vacíos ya que en ese caso el peso total sería < 2 pues:

18

Page 19: Invariantes. Todo lo que debías saber para la OMA

1. Las piedras en la primera columna a lo sumo suman2 14

+ 18

+ 116

+ .... = 12

2. Las piedras en la segunda columna a lo sumo suman 14

+ 18

+ 116

+ .... = 12

3. Las piedras en la tercera columna a lo sumo suman 14

+ 18

+ 116

+ .... = 12

4. Las piedras en la cuarta columna a lo sumo suman 18

+ 116

+ 132

+ .... = 14

5. Las piedras en la quinta columna a lo sumo suman 116

+ 132

+ 164

+ .... = 18

6. .......

Es decir que entre todas las piedras a lo sumo suman

1

2+

1

2+

1

2+

1

4+

1

8+

1

16+ .... = 1 + 1 = 2

Pero para darse la igualdad deberíamos usar TODOS los puntos por lo que nece-sitaríamos infinitas piedras e infinitos turnos. ¡Absurdo! De donde no es posible dejarvacíos los puntos (0, 0), (0, 1) y (1, 0). ‡

Algunos problemas para pensar

1. Tenemos el mismo problema que antes pero con sólo 1 piedra en la casilla (0, 0).El objetivo es dejar vacías las casillas (0, 0), (0, 1), (1, 0), (0, 2), (1, 1), (2, 0). ¿Esposible lograrlo? (Ésta es la segunda parte del problema original, la primera eracon las 6 casillas ocupadas)

2. (Moscú 1995) Tenemos 4 triángulos rectángulos iguales. En cada turno, pode-mos elegir cualquier triángulo de los que tenemos y cortarlo en 2 por la alturadel triángulo rectángulo (notar que quedan formados 2 triángulos rectángulos).Probar que siempre vamos a tener 2 triángulos iguales.

3. Se tiene una tira infinita de casilleros con algunas piedras distribuidas entre ellos(puede haber más de una piedra por casillero). En cada turno se pueden quitar2 piedras del mismo casillero y agregar una piedra al casillero inmediatamentea la derecha. Probar que sin importar como procedamos, llega un momento enque no podemos seguir moviendo piedras y la distribución final es independientede nuestras elecciones.

2Para sumas de este tipo consultar el apéndice “Sumas infinitas“

19

Page 20: Invariantes. Todo lo que debías saber para la OMA

2.5. Fichas

A veces resulta necesario llevar a cabo un procedimiento complejo para elaborarun invariante, pero el solo hecho de proponerse la construcción del mismo puede llevara que lo hallemos.

Lo que sigue es un caso particular de un problema de la Olimpíada Rusa de 1997.El problema gustó tanto que fue votado como “El mejor Problema del año“ por losparticipantes.

Consideremos una tira infinita de casilleros numerados con los númerosenteros. En algunos casilleros hay fichas de forma que la cantidad total defichas es finita (puede haber casilleros con 2 o más fichas). En cada pasose pueden realizar 2 tipos de jugadas

1. Si hay 2 casillas no vacías seguidas, n y n + 1, se saca una de cadauna y se pone una ficha en la casilla n + 2.

2. Si hay al menos 2 fichas en la casilla n, se retiran las 2 fichas y sepone una en la casilla n + 1 y otra en la casilla n− 2.

Demostrar que toda secuencia de movidas conduce irremediablemente auna posición desde la cual es imposible realizar nuevas movidas y sinimportar las jugadas que se hicieron siempre se llega a la misma posición.

Lo primero que haremos en este problema es buscar un invariante. La idea esescribir en cada casilla un número que representará el peso de cada ficha en esa casillade forma tal que la suma de los pesos de todas las fichas (el “peso total“) no varíe alrealizar las operaciones permitidas. Veamos entonces cómo escribir los números.

Si en la “n−ésima“ casilla escribimos el número an (notar que n puede ser negativo)y cualquier operación deja invariante el peso total, entonces en cada paso la suma delos pesos de las fichas que retiramos debe ser lo mismo que la suma de los pesos delas fichas que agregamos. Esto se traduce en que para cada n:

1. an+2 = an+1 + an

2. 2an = an+1 + an−2

Cuando se tienen sucesiones con este tipo de relaciones3 es común poner an = αn

para todo n y las condiciones se transforman en:

3Mirar el apéndice “Sucesiones recurrentes“

20

Page 21: Invariantes. Todo lo que debías saber para la OMA

1. αn+2 = αn+1 + αn

2. 2αn = αn+1 + αn−2

Sacando factor común αn en la primer ecuación y αn−2 en la segunda llegamos a:

1. α2 = α + 1

2. 2α2 = α3 + 1

La ventaja de hacer el reemplazo anterior es que de esta forma las condiciones yano dependen de n. Antes teníamos infinitas condiciones y ahora solamente dos. No esdifícil ver que α = 1+

√5

2, que es la raíz positiva de x2 − x− 1 = 0, satisface ambas.

Pongamos entonces α = 1+√

52

, escribamos para cada n el número αn en la n−ésimacasilla y decretemos que el peso de una ficha es el número escrito en la casilla queocupa. Como para todo n, αn+2 = αn+1 + αn y 2αn = αn+1 + αn−2 entonces en cadapaso, sin importar qué operación realicemos, la suma de los pesos de las fichas novaría. ¡Hemos encontrado el invariante!

Probemos que toda configuración inicial T con peso total w(T ) conduce a unaposición en donde no se pueden realizar más movimientos. Como α > 1, debe existirk0 tal que αk > w(T ) para todo k ≥ k0. Al ser todos los pesos positivos nuncapodremos poner una ficha en la casilla k0 o a su derecha ya que ésta sola superaría elpeso total. Podemos concluir entonces que hay una ficha que llegó más lejos hacia laderecha que cualquier otra. Al hacerlo, esta ficha debe quedarse quieta por el resto deljuego pues si se mueve tendríamos que poner una ficha a su derecha lo que contradicesu elección. Clavemos esta ficha y repitamos el argumento con el resto (haciendo decuenta que la ficha clavada no existe). Continuando de esta manera, como nunca secrean fichas, a lo sumo se descartan, tarde o temprano todas las fichas deben estarclavadas y no se podrán mover más fichas.

Ahora debemos probar que sin importar el orden en que se hicieron las jugadassiempre se llega a la misma posición. En caso contrario tendríamos 2 distribucionesdistintas de fichas Ti y Tj, sin 2 en la misma casilla ni 2 en casillas consecutivas, talesque w(Ti) = w(Tj). Veamos que esto es imposible, para ello podemos suponer queninguna ficha de Ti está en la misma casilla que una de Tj, pues en ese caso quitamosesas fichas y tenemos un contraejemplo más chico. Si las fichas de Ti están en lascasillas i1 < i2 < ... < in (con ik−1 ≤ ik − 2) y las de Tj en j1 < j2 < ... < jm (conjk−1 ≤ jk − 2) luego4:

w(Ti) = αin + αin−1 + .... + αi1

4Consultar el apéndice “Sumas infinitas“

21

Page 22: Invariantes. Todo lo que debías saber para la OMA

w(Ti) < αin + αin−2 + αin−4 + ....

w(Ti) < αin(1 + α−2 + α−4 + ...)

w(Ti) < αin1

1− α−2= αin

α2

(α2 − 1)

w(Ti) < αinα2

α= αin+1

Luego αin+1 > w(Ti) = w(Tj) ≥ αjm , es decir que in +1 > jm o lo que es lo mismoque in ≥ jm, pero de forma análoga jm ≥ in y podemos concluir que in = jm. Es decirque Ti tiene una ficha en la misma casilla que Tj tiene otra, lo que contradice nuestrasuposición. Entonces cualesquiera 2 distribuciones desde las que no se puedan realizarmovidas con el mismo peso total deben ser iguales.

Hemos probado entonces que no importa qué las elecciones que tomemos, siem-pre se llega a una configuración desde donde no es posible mover más fichas y estaconfiguración siempre es la misma. ‡

En el problema anterior queríamos asignar números de forma tal que determinadacuenta sencilla permaneciera invariante a las operaciones del problema; los que noresultarían ser tan sencillos son los números que debíamos encontrar para que secumpliera eso, pero el hecho es que al plantear las condiciones estos aparecieron deforma natural.

Algunos problemas para pensar

1. ¿Qué pasa si en la solución anterior en vez de poner an = αn ponemos an = Fn

donde Fn es el n−ésimo término de la sucesión de Fibonacci definida recursiva-mente por F0 = 0, F1 = 1 y Fn+2 = Fn+1 + Fn?

2. (Rusia 1997) Sea k ≥ 2 un número natural. Consideremos una tira infinita decasilleros numerados con los números enteros. En algunos casilleros hay fichasde forma que la cantidad total de fichas es finita (puede haber casilleros con 2o más fichas). En cada paso se pueden realizar 2 tipos de jugadas

a) Si hay k casillas no vacías seguidas, n, n+1, ..., n+k− 1, se saca una fichade cada una y se pone una ficha en la casilla n + k.

b) Si hay al menos 2 fichas en la casilla n, se retiran las 2 fichas y se poneuna en la casilla n + 1 y otra en la casilla n− k.

Demostrar que toda secuencia de movidas conduce irremediablemente a unaposición desde la cual es imposible realizar nuevas movidas y sin importar lasjugadas que se hicieron siempre se llega a la misma posición.

22

Page 23: Invariantes. Todo lo que debías saber para la OMA

3. Sea {Fn} la sucesión de Fibonacci definida recursivamente por F0 = 0, F1 = 1y Fn+2 = Fn+1 + Fn. Probar que todo numero natural n > 0 tiene una únicaexpresión de la forma

n = Fi1 + Fi2 + ..... + Fik

donde i1 ≥ 2 y ij+1 ≥ ij + 2 para todo j.

4. (Apmo 2006) Probar que todo entero positivo se puede escribir como la sumafinita de algunas potencias distintas de el número de oro. Es decir, si α =1+

√5

2entonces para todo n entero positivo, existen i1, ..., ik enteros distintos no

necesariamente positivos tales que

n = αi1 + ........ + αik

5. (Imo 1993) Construir una función f : N → N tal que f(1) = 2 y para todon ∈ N

f(f(n)) = f(n) + n

f(n) < f(n + 1)

6. Se tiene una pila de n fichas que forman una columna vertical. Esta configuraciónse modifica de acuerdo a las siguientes reglas. Una ficha se puede mover siestá en el tope de una columna que contiene por lo menos dos fichas más quela columna que está inmediatamente a la derecha (si no hay columnas a laderecha, considerar que hay columnas con 0 fichas). En cada etapa, se eligeuna de las fichas que se pueden mover (si es que hay alguna) y se coloca enel tope de la columna que está a su derecha. Si no se puede mover ningunaficha la configuración se llama final. Para cada n, demostrar que no importaqué elecciones se tomen en cada etapa, la configuración final que se obtiene esúnica. Describir dicha configuración en términos de n.

23

Page 24: Invariantes. Todo lo que debías saber para la OMA

2.6. Un poco de Geometría Proyectiva

Dado un punto o y una recta L que no pasa por o, la proyección en la recta L deun punto a desde o es el punto a′ que se obtiene como la intersección de ao y L 5.

Inicialmente tenemos 4 puntos alineados a0, b0, c0, d0 (en ese orden) talesque a0b0 = b0c0 = c0d0 = 1. En cada turno podemos elegir una recta yproyectar los 4 puntos en la recta desde cualquier punto, luego borramoslos 4 puntos que proyectamos y nos quedamos con los nuevos. ¿Es posibleque luego de varios turnos obtengamos 4 puntos alineados a1, b1, c1, d1 (enese orden) tales que a1b1 = 2b1c1 = c1d1 = 2?

Supongamos que es posible y lleguemos a un absurdo. Para ello vamos a demostrarque la razón doble de los 4 puntos permanece invariante. Si tenemos 4 puntos a, b, c, dsu razón doble se denota por {ac : bd} y se define como:

{ac : bd} =ab

cb/ad

cd=

ab · cdcb · ad

Consideremos ahora un punto o y proyectemos 4 puntos a, b, c, d de una recta Len otra L′, obteniendo a′, b′, c′, d′. Debemos probar que {ac : bd} = {a′c′ : b′d′}.

Sea p la intersección de L y L′. Usando el Teorema de Menealo en los triángulosapa′ y cpc′ con la recta bb′o obtenemos:

pb · a′b′

ab · pb′=

a′o

ao(1)

pb · c′b′

bc · pb′=

c′o

co(2)

5Si ao//L decimos que a se fue al punto del infinito de L o que se fue al infinito en la direcciónde L.

24

Page 25: Invariantes. Todo lo que debías saber para la OMA

Ahora volvamos a usar el Teorema de Menelao en los triángulos apa′ y cpc′ perocon la recta dd′o.

pd · a′d′

ad · pd′=

a′o

ao(3)

pd · c′d′

dc · pd′=

c′o

co(4)

Luego

pb · a′b′

ab · pb′pd · c′d′

dc · pd′=

pb · c′b′

bc · pb′pd · a′d′

ad · pd′

De donde

a′b′ · c′d′

ab · dc=

c′b′ · a′d′

bc · ada′b′ · c′d′

c′b′ · a′d′=

ab · dc

bc · ad{ac : bd} = {a′c′ : b′d′}

Por lo recién visto, si fuera posible empezar con a0, b0, c0, d0 y terminar cona1, b1, c1, d1 entonces:

{a0c0 : b0d0} = {a1c1 : b1d1}

Pero esto es absurdo pues

{a0c0 : b0d0} =a0b0 d0c0

b0c0 a0d0

=1 · 11 · 3

=1

3

{a1c1 : b1d1} =a1b1 d1c1

b1c1 a1d1

=2 · 21 · 5

=4

5

Entonces no es posible lo que pide el enunciado. ‡

Lo anterior es un ejemplo de algo muy usado en geometría; “Las Transformaciones“.Veamos otro ejemplo. Imaginemos un punto o y 2 planos π y π′. La proyección de πsobre π′ desde o es a cada punto p en π mandarlo a la intersección p′ de op con π′ 6.¿Qué preserva la proyección?

6Si op//π′ entonces p′ es el punto del infinito de π′ en la dirección de L.

25

Page 26: Invariantes. Todo lo que debías saber para la OMA

La misma cuenta que hicimos en la solución anterior demuestra que la razóndoble de puntos alineados es invariante en este tipo de proyección; pero hay otraestructura más simple que se preserva. Notemos que si a, b, c están en π y a′, b′, c′ sonsus proyecciones en π′ entonces a, b, c están alineados si y sólo si a′, b′, c′ lo están. Dichode otra forma, la proyección manda “rectas en rectas“. Más aún si P, Q, R son 3 rectasen π y P ′, Q′, R′ son sus proyecciones sobre π′ entonces P, Q,R son concurrentes si ysólo si P ′, Q′, R′ lo son7.

De ahora en adelante, cuando tengamos un objeto T en π su proyección T ′ es elconjunto de puntos de π′ que son las proyecciones de puntos de T . La proyección deun círculo no es necesariamente un círculo. Lo que ocurre es que no estamos mirandola propiedad correcta, el círculo es miembro de una familia más grande de curvas: “lascónicas“ 8. Los 4 miembros de esta familia son los círculos, las parábolas, las elipsesy las hipérbolas. La proyección de una cónica es una cónica. Es decir “ser un círculo“no es invariante por proyección, pero sí lo es “ser una cónica“.

Otra propiedad invariante por proyección es “ser tangente“ a una cónica. Es decirsi tenemos una recta L y una cónica ω y L′ y ω′ son sus proyecciones entonces Les tangente a ω si y sólo si L′ es tangente a ω′. Dado un punto p en el exterior deuna cónica, ¿será posible trazar las tangentes a la cónica usando sólo una regla? Porejemplo para un círculo podemos proceder de la siguiente forma:

Dado un círculo ω y un punto p en su exterior tomemos 2 puntos arbi-trarios a y b en ω y tracemos las rectas pa y pb que vuelven a cortar a ωen c y d respectivamente. Tracemos ahora las rectas ab y cd que se cortanen q y ad y bc que se cortan en r. Por último tracemos qr que corta a ωen s y t. Luego ps y pt son las tangentes a ω por p.

7¡Cuidado! si P y Q son 2 rectas paralelas, P ′ y Q′ no necesariamente lo son.8Cada cónica ω se puede pensar como la intersección de un cono con un plano.

26

Page 27: Invariantes. Todo lo que debías saber para la OMA

Ahora, como la construcción entera es invariante por proyección, el mismo proced-imiento funciona para trazar las tangentes a una cónica. Es decir, supongamos querealizamos la construcción anterior para una cónica ω y queremos probar que pr y qrresultan tangentes a ω. Proyectemos el dibujo a una plano π′ de forma que la proyec-ción ω′ de ω resulte un círculo (¿Se les ocurre cómo hacer esto?). Ahora tenemos unpunto p′ y 2 rectas por él que cortan a ω′ en a′ y c′ la primera y b′ y d′ la segunda.La intersección de a′b′ y c′d′ es q′ y la intersección de a′d′ y b′c′ es r′. Por último, q′r′

corta a ω′ en s′ y t′. Como esto no es otra cosa que la construcción anterior aplicadaa ω′ y p′, resulta que p′s′ y p′t′ son tangentes a ω′, de donde proyectando de vuelta,obtenemos que ps y pt son tangentes a ω.

¿Será posible usando sólo una regla marcar el centro de un círculo? Másprecisamente, lo que preguntamos es si existe un procedimiento que dadocualquier círculo nos permita encontrar su centro usando sólo una regla(como el anterior para trazar las tangentes a un círculo por un punto ensu exterior). Con la regla lo que podemos hacer es lo siguiente

1. Trazar la recta que une 2 puntos ya marcados.

2. Marcar la intersección de 2 rectas ya dibujadas.

3. Trazar una recta arbitraria por un punto dado.

4. Marcar un punto arbitrario sobre el círculo original o sobre una rectaya dibujada.

No, no es posible. Supongamos que sí, tenemos entonces un procedimiento quepara cualquier círculo nos permite encontrar su centro usando sólo una regla como en1,2,3 o 4.

27

Page 28: Invariantes. Todo lo que debías saber para la OMA

Imaginemos 2 planos π y π′ y 2 círculos ω y ω′ de forma que ω está en π y ω′

es la proyección de ω sobre π′. Apliquemos nuestro procedimiento para determinar elcentro de ω de forma que cada cosa que vamos dibujando en π la vamos proyectandoa π′. ¿Qué podemos decir de lo dibujado en π′? Resulta que si alguien viene y observaπ′ sin saber lo que estamos haciendo en π creerá que estamos aplicando nuestroprocedimiento para determinar el centro de ω′ pues al proyectar lo que hacemos en πes como si estuviéramos haciéndolo en π′. Por ejemplo, si tenemos a y b en π, trazamosla recta L que los une y la proyectamos obteniendo L′, es como si directamentehubiéramos trazado la recta que une a′ con b′. Si marcamos la intersección p de 2rectas L1 y L2 y la proyectamos a π′ obteniendo p′ es como si hubiéramos marcadola intersección de L′

1 y L′2.

Como nuestra construcción sirve para cualquier círculo independientemente denuestras elecciones (que vienen de usar la regla en cosas como en 3 y 4) entoncescuando encontremos el centro de ω y lo proyectemos hasta π′, éste será el centro deω, pues el dibujo en π′ es nuestro procedimiento aplicado para determinar el centrode ω′. Dicho de otra forma.. ¡Nuestro procedimiento es invariante por proyección!

El problema es que si bien nuestro procedimiento es invariante por proyección, nolo es la propiedad “ser el centro de un círculo“. En realidad, como dijimos antes nisiquiera “ser un círculo“ lo es, de forma que debemos tener un poco de cuidado.

Para terminar el problema lo que hacemos es elegir los planos π y π′ y los círculosω y ω′ de forma que la proyección del centro de ω no sea el centro de ω′ 9, lo quecontradice nuestra conclusión anterior. Entonces no puede existir un procedimientoque nos permita encontrar el centro de un círculo usando sólo una regla. ‡

La demostración anterior merece algunos comentarios. En primer lugar, el prob-lema anterior no es un problema común. Una cosa es encontrar un procedimientopara hacer algo pero otra dramáticamente distinta es probar que tal procedimientono puede existir. Ejemplos de esta situación son la cuadratura del círculo, la construc-ción de algunos polígonos regulares con regla y compás o la resolución de ecuacionesde grado ≥ 5. Probar que no existe ninguna fórmula para encontrar las raíces deuna ecuación de grado 5 precisó (y motivó) del desarrollo de uno de los trabajos másprofundos, bellos e importantes de la matemática: “La Teoría de Galois“. En segundolugar, hay algo en la demostración anterior que la hace “especial“, el problema hablabade algo que ocurre en el plano pero la demostración es en 3 dimensiones. Otro ejemplode cuan importante es no encasillar el pensamiento de uno, de no limitarlo, de dejarvolar la imaginación.

9Mirar el problema 11

28

Page 29: Invariantes. Todo lo que debías saber para la OMA

Algunos problemas para pensar

1. Dado un punto o, un ángulo α y una constante k consideremos la siguientetransformación del plano en sí mismo. A cada punto p lo mandamos a p′ deforma que pop′ = α y po

p′o= k (a mí me gusta pensarla como que tenemos un

triángulo T y cada punto p lo mandamos a p′ de forma que pop′ sea semejante aT ). Probar que preserva colinealidad, ángulos entre rectas y razón de segmentos.

2. Dado un punto o y una constante k, consideremos la siguiente transformación delplano en sí mismo. A cada punto p mandémoslo a un punto p′ en la semirrectaop tal que op×op′ = k2. Probar que la transformación manda círculos en círculos(para este problema considerar las rectas como círculos de radio infinito). Probarademás que

a′b′ =ab · k2

oa · obDeducir que preserva razón doble.

3. Dado un cuadrilátero convexo abcd probar que los 4 vértices están en una cir-cunferencia o en una recta si y sólo si

ab · cd + ad · bc = ac + bd

4. Dado un triángulo abc y un círculo ω que corta a ab en d y e, a bc en f y g y aca en h e i sean p, q y r las intersecciones de las tangentes al círculo en d y e, fy g y en h e i. Probar que aq, br y cp son concurrentes.

5. Dado un punto o en una circunferencia ω y una recta L definimos la proyecciónde ω en L como la transformación que a cada punto p en ω lo manda a p′ en Lque es la intersección de op con L. Probar que preserva razón doble.

6. Una transformación de un círculo o una recta en otro círculo o recta se diceproyectiva si se obtiene como composición de proyecciones (por ejemplo primeroproyectamos a un círculo, luego a una recta (no necesariamente desde el mismopunto), luego a otra recta y así). Probar que las transformaciones proyectivaspreservan razón doble.

7. Consideremos 2 planos π y π′ y una recta L que no es paralela a ninguno de los 2planos. A cada punto p en π lo mandamos un punto p′ en π′ que es la intersecciónde una recta paralela a L por p con π′ (es como si proyectáramos desde un puntoen el infinito en la dirección de L). Probar que esta transformación preservacolinealidad, concurrencia, razón de segmentos paralelos y razones de áreas.

29

Page 30: Invariantes. Todo lo que debías saber para la OMA

8. Probar que dado un triángulo T y 3 puntos a, b y c en un plano π podemostransformarlos mediante una transformación como la del problema anterior ena′, b′, c′ en un plano π′ de forma que a′b′c′ sea semejante a T .

9. Sea abcde un pentágono convexo y consideremos f = bc ∩ de, g = cd ∩ ea,h = de ∩ ab, i = ea ∩ bc y j = ab ∩ cd. Probar que si las áreas de los triángu-los ahi, bij, cjf, dfg, egh son todas iguales entonces af, bg, ch, di, ej son concur-rentes.

Sugerencia: Proyectar de forma que a′c′d′ vaya a parar a los vértices de unpentágono regular.

10. Probar que no es posible usando sólo una regla trazar una paralela a un segmentodado.

11. Supongamos que tenemos una esfera S y un plano π′ tangente a S en a. Seao el punto de S diametralmente opuesto a a. Tomemos un plano π que pasepor el centro de S pero que no sea paralelo a π′ y llamemos ω al círculo quees la intersección de S con π. Por último tomemos p en π′ de forma que op seaperpendicular a π. Probar que ω′, la proyección de ω a π′ desde o, es un círculode centro p.

30

Page 31: Invariantes. Todo lo que debías saber para la OMA

2.7. Cartas

Esta vez el problema es de la lista corta de la 46 IMO del 2005.

Se tienen n cartas alineadas en una fila, cada una de ellas con un ladoblanco y el otro negro, todas con su lado blanco hacia arriba. En cadaetapa, si es posible, se elige una carta con el lado blanco hacia arriba (queno está en el borde de la fila), se quita y se dan vuelta la carta más próximahacia la derecha y la carta más próxima hacia la izquierda. Demostrar quese puede llegar a una situación en la que sólo quedan dos cartas si y sólosi n− 1 no es divisible por 3.

En primer lugar veamos que si n−1 no es divisible por 3 entonces es posible dejarsólo 2 cartas. Notemos que si primero sacamos la segunda carta, luego la cuarta y porúltimo la tercera entonces volvemos a la situación original pero con 3 cartas menos.

BBBBB....... → NNBB....... → NBN....... → BB.......

Luego, si n − 1 no es divisible por 3, repitiendo lo anterior vamos sacando de a3 cartas hasta llegar a BB o a BBB. Si llegamos a BB ya estamos y si llegamos aBBB entonces quitamos la del medio y estamos en NN ; en ambos casos logramosdejar sólo 2 cartas, como queríamos.

Para ver que si n− 1 es divisible por 3 entonces no es posible dejar sólo 2 cartasvamos a usar un invariante. Esta vez va a ser necesario de una construcción auxiliarpara entenderlo mejor.

Vamos a pensar cada carta como una orden. Dados 3 números ordenados, lascartas negras intercambian los 2 últimos y las blancas mueven cada número un lugara la derecha (es decir que el primero toma el lugar del segundo, el segundo el deltercero y el tercero el del primero).

N(xyz) = xzy

B(xyz) = zxy

Las órdenes se leen de derecha a izquierda, por ejemplo BBN(123) = 321 pues

N(123) = 132

B(132) = 213

B(213) = 321

31

Page 32: Invariantes. Todo lo que debías saber para la OMA

O en la notación que vamos a usar

BBN(123) = BB(N(123)) = BB(132) = B(B(132)) = B(213) = 321

No es difícil ver que tenemos las siguientes relaciones

BBB = NN (1)BBN = NB (2)NBB = BN (3)NBN = BB (4)

Por ejemplo

NBN(xyz) = NB(xzy) = N(yxz) = yzx

BB(xyz) = B(zxy) = yzx

Además BBB(xyz) = NN(xyz) = xyz.

Volviendo al problema, consideremos las siguientes jugadas. Empezamos con 5cartas blancas; primero quitamos la segunda carta (de izquierda a derecha), luego lacuarta y por último la tercera. La situación la podemos representar como sigue:

BBBBB → NNBB → NBN → BB

¿Qué tienen en común BBBBB,NNBB, NBN y BB? Para poder responderesta pregunta es que precisamos pensar a las cartas como órdenes que se aplican a123. Las 4 secuencias tienen en común que pensadas como un conjunto de órdenesaplicadas a 123 dan lo mismo, es decir:

BBBBB(123) = 231

NNBB(123) = 231

NBN(123) = 231

BB(123) = 231

Es claro ahora cuál será el invariante, hay que pensar a las cartas como órdenesy aplicarlas (de derecha a izquierda) a 123. Para ello, debemos verificar que en cadapaso el resultado final no cambia. Supongamos por ejemplo que sacamos una cartablanca con 2 vecinas negras (los demás casos se deducen de la misma forma de lasrelaciones (1), (2) y (3)), entonces debemos ver que

...NBN...(123) = ...BB...(123)

32

Page 33: Invariantes. Todo lo que debías saber para la OMA

Pero de (4) sabemos que NBN = BB luego

...NBN...(123) = ...NBN(...(123))

...NBN...(123) = ...(NBN(...(123)))

...NBN...(123) = ...(BB(...(123)))

...NBN...(123) = ...BB(...(123))

...NBN...(123) = ...BB...(123)

Como queríamos.

Para terminar el problema, como BBB(123) = 123, solamente hace falta notarque si n = 3k + 1 entonces

BBBB......BBB(123) = BBBB......(123) = BBBB(123) = B(123) = 312

De donde no es posible que queden 2 cartas pues

BB(123) = B(312) = 231

NB(123) = N(312) = 321

BN(123) = B(132) = 213

NN(123) = N(132) = 123

es decir que ninguno coincide con 312. ‡

La demostración anterior se puede hacer más gráfica pensando que 123 son losvértices de un triángulo equilátero. Cada vez que tenemos una B lo rotamos 120o ensentido antihorario y cada vez que tenemos una N lo reflejamos por la altura vertical(es decir que intercambiamos los 2 vértices de abajo).

33

Page 34: Invariantes. Todo lo que debías saber para la OMA

Completemos ahora la construcción del problema, así como definimos N(xyz) =xzy y B(xyz) = zxy podríamos haber definido otras órdenes...

I(xyz) = xyz N1(xyz) = xzy

B1(xyz) = zxy N2(xyz) = zyx

B2(xyz) = yzx N3(xyz) = yxz

Ahora por ejemplo B1N1N2(xyz) = yzx pues

B1N1N2(xyz) = B1N1(zyx) = B1(zxy) = yzx

De la misma forma que antes, tenemos varias relaciones, por ejemplo B1N2 = N2 yaque

B1N2(xyz) = B1(zyx) = xzy = N1(xyz)

Análogamente se pueden deducir las siguientes relaciones (o reglas para reemplazar2 órdenes por 1 sola):

II = I IB1 = B1 IB2 = B2 IN1 = N1 IN2 = N2 IN3 = N3

B1I = B1 B1B1 = B2 B1B2 = I B1N1 = N3 B1N2 = N1 B1N3 = N2

B2I = B2 B2B1 = I B2B2 = B1 B2N1 = N2 B2N2 = N3 B2N3 = N1

N1I = N1 N1B1 = N2 N1B2 = N3 N1N1 = I N1N2 = B1 N1N3 = B2

N2I = N2 N2B1 = N3 N2B2 = N1 N2N1 = B2 N2N2 = I N2N3 = B1

N3I = N3 N3B1 = N1 N3B2 = N2 N3N1 = B1 N3N2 = B2 N3N3 = I

No hay dificultad ahora en resolver el siguiente problema:

Se tiene en el pizarrón una secuencia de letras. En cada turno, se puedenelegir 2 letras, borrarlas y escribir otra de acuerdo a las siguientes reglas

AA → A AB → B AC → C AD → D AE → E AF → F

BA → B BB → C BC → A BD → F BE → D BF → E

CA → C CB → A CC → B CD → E CE → F CF → D

DA → D DB → E DC → F DD → A DE → B DF → C

EA → E EB → F EC → D ED → C EE → A EF → B

FA → F FB → D FC → E FD → B FE → C FF → A

Probar que sin importar las elecciones que tomemos, la última letra quequeda es siempre la misma.

34

Page 35: Invariantes. Todo lo que debías saber para la OMA

Algunos problemas para pensar

1. En el problema de las cartas digamos que el peso de una carta blanca es (−1)k

donde k es la cantidad de cartas negras a su izquierda. Probar que en cada etapa,la suma de los pesos de las cartas blancas aumenta o disminuye en una cantidadmúltiplo de 3 o se mantiene igual. Usar esto para dar otra demostración.

2. En un pizarrón hay cierta cantidad de letras e, a, b, c. En cada turno se puedenborrar 2 letras y escribir otra en función de la siguiente tabla

e a b ce e a b ca a e c bb b c e ac c b a e

Demostrar que sin importar lo que hagamos, la última letra siempre es la misma.

Sugerencia: Poner E(wxyz) = wxyz, A(wxyz) = xwzy, B(wxyz) = yzwx yC(wxyz) = zyxw (se puede pensar que si xyzw es un cuadrado entonces A yB es reflejarlo por sus bases medias y que C es reflejarlo por su centro).

3. (XIV Olimpíada Rioplatense) Se desea colorear cada entero positivo con uncolor utilizando la mayor cantidad posible de colores de manera que se verifiquela siguiente condición: Si, en notación decimal, el número B se puede obtenera partir del número A, suprimiéndole a A dos dígitos iguales consecutivos (aa)o suprimiéndole a A cuatro dígitos consecutivos que formen dos pares iguales yconsecutivas (abab), entonces A y B son del mismo color.

Por ejemplo, 8, 833 y 22811 deben ser del mismo color y también 72, 676772 y1173329898 son del mismo color.

Determinar cuál es la mayor cantidad de colores que se puede utilizar.

35

Page 36: Invariantes. Todo lo que debías saber para la OMA

2.8. Rompecabezas

Ahora es el turno de un juego que ya era conocido en 1870. El rompecabezas,popularizado como “Fifteen Puzzle“, es de Noyes Chapman10.

Se tiene un rompecabezas con forma de un tablero de 4× 4 con 15 fichasnumeradas 1, 2, ....., 15. Siempre debe haber una casilla vacía y el restoocupadas por una ficha cada una. En cada turno podemos elegir unacasilla vecina a la vacía y mover su ficha hasta ésta. ¿Es posible llegar dela posición de la izquierda a la de la derecha?

1 2 3 45 6 7 89 10 11 1213 14 15

1 2 3 45 6 7 89 10 11 1213 15 14

Sea P0 la posición inicial y Pn a la que queremos llegar. Supongamos que existenposiciones intermedias P1, ..., Pn−1 de forma que cada una se obtiene de la anteriorempujando una ficha a la casilla vacía. Es fácil ver que n debe ser par. Para ello,pintemos las casillas de blanco y negro como las de un tablero de ajedrez y notemosque la casilla vacía siempre cambia de color. Como P0 y Pn tienen la misma casillavacía entonces al final del juego, ésta debe haber cambiado de color una cantidad parde veces o lo que es lo mismo n debe ser par.

a1 a2 a3 a4

a5 a6 a7 a8

a9 a10 a11 a12

a13 a14 a15 a16

Podemos pensar que la casilla vacía está ocupada por una ficha invisible con un16 y en cada turno podemos intercambiar esta ficha con cualquier vecina. Definamospara una posición P , con las fichas como en la figura anterior, su signo como:

sg P = sg∏i<j

(aj − ai)

Recordemos que sg a = 1 si a > 0 y sg a = −1 si a < 0 (el producto nunca es ceroya que todos los ai son distintos entre sí). Vamos a usar el signo para llegar a unabsurdo. Analicemos qué pasa con el signo después de cada turno.

10Aunque también algunos dicen que pertenece a Sam Lloyd

36

Page 37: Invariantes. Todo lo que debías saber para la OMA

Supongamos que Pm+1 se obtiene de Pm al intercambiar ai con ai+k (es decir queai = 16 o ai+k = 16, además por la forma del tablero k = 1 o k = 4). Para calcularsg Pm+1 debemos calcular el signo de el producto de todas las diferencias de losnúmeros en Pm+1. Este producto es muy parecido al que debemos calcular para Pm,la única diferencia es que los siguientes factores se dan vuelta:

ai+1 − ai, ai+2 − ai, ...., ai+k−1 − ai, ai+k − ai, ai+k − ai+k−1, ...., ai+k − ai+1

Como 2k − 1 factores cambiaron por su opuesto, entonces

sg Pm+1 = (−1)2k−1sg Pm = −sg Pm

De donde, recordando que n es par

sg Pn = −sg Pn−1 = sg Pn−2 = ... = −sg P1 = sg P0

Es decir que sg Pn = sg P0. Pero es fácil ver que sg P0 = 1 y sg Pn = −1, lo que esabsurdo que viene de suponer que era posible llegar a la posición deseada. ‡

Notemos que si bien el signo no es un invariante, éste varía de forma controlada.Si queremos un invariante, para cada posición P pongamos I(P ) = sg P t(P ) dondet(P ) = 1 si la casilla vacía está en una casilla negra y t(P ) = −1 si está en una blanca.

Algunos problemas para pensar

1. Supongamos que tenemos 2 posiciones para el rompecabezas del problema conel mismo signo y la casilla vacía en el mismo lugar. Probar que se puede llegarde una a la otra mediante las operaciones permitidas.

2. (Olimpíada Balkánica 2001) Se tiene un cubo de 3× 3× 3 dividido en 27 celdasunitarias. Una de las celdas está vacía y el resto está ocupada por cubitosunitarios numerados 1, 2, ..., 26. Cada jugada consiste en empujar un cubitounitario a una celda vecina (que comparte una cara), si ésta se encuentra vacía.¿Es posible que después de algunas operaciones cada cubito numerado k estéen el lugar donde empezó el cubito 27− k?

3. El Cubo de Rubik es un cubo de 3× 3× 3 dividido en 27 cubitos unitarios concada cara del cubo grande pintada de un color distinto. En cada turno podemosrotar 90o una cara a elección (es decir los 9 cubitos que la componen). ¿Esposible que después de algunos turnos volvamos a la posición original salvo porun único cubito unitario que intercambió dos de sus caras de lugar?

37

Page 38: Invariantes. Todo lo que debías saber para la OMA

4. (Lista Corta 40 IMO) Hay n chicos y cada uno tiene una pelota con su nombre.En cierto momento los chicos comienzan a intercambiarse las pelotas, es decir Ase la da a B y B se la da a A. Se sabe que al final del día cada chico se fue consu pelota y que cada par de chicos intercambiaron 1 vez la pelota exactamente.Hallar los posibles valores de n para los que esto es posible.

5. Supongamos que tenemos n objetos, cada uno de ellos con una etiqueta. En cadaturno podemos intercambiar la etiqueta de cualesquiera 2 objetos. Demostrarque es imposible que después de una cantidad impar de turnos cada objetotenga su etiqueta.

6. Dada una permutación11 π de 1, 2..., n definimos

sg (π) = sg∏i<j

(π(j)− π(i)) =∏i<j

π(j)− π(i)

j − i

Probar que sg (π1 ◦ π2) = sg (π1) sg (π2)

7. Es dado un tablero de n×n con n impar, que está cubierto con fichas de dominó,salvo por una esquina. Las fichas de dominó se pueden deslizar por el tablero sila casilla nueva a ocupar está vacía. ¿A qué casillas podemos trasladar la casillavacía?

11Consultar el apéndice de Permutaciones para la notación.

38

Page 39: Invariantes. Todo lo que debías saber para la OMA

2.9. Pizarrones, Tablas y Grupos

Es momento de volver sobre algunos problemas

En un pizarrón hay 10 signos + y 5 signos −. En cada turno se debeelegir dos de ellos, borrarlos y poner un + si eran iguales y un − si erandistintos. Probar que después de 14 turnos quedará un signo − solamente.

En un pizarrón hay cierta cantidad de letras a, b, c. En cada turno se puedereemplazar una a y una b por una b, una a y una c por una c, una b y unac por una a, dos a por una a, dos b por una c o dos c por una b. El objetivoes dejar una sola letra. Probar que sin importar en qué orden realicemoslas operaciones, la letra que quede al final siempre será la misma.

La información en los problemas anteriores se puede resumir en las siguientestablas

+ −+ + −− − +

a b ca a b cb b c ac c a b

Hagamos ahora el camino inverso y reconstruyamos el problema a partir de latabla.

En un pizarrón hay cierta cantidad de letras e, a, b, c. En cada turno sepueden borrar 2 letras y escribir otra en función de la siguiente tabla

e a b ce e a b ca a e c bb b c e ac c b a e

Demostrar que sin importar lo que hagamos, la última letra siempre es lamisma.

39

Page 40: Invariantes. Todo lo que debías saber para la OMA

Si ponemos un 1 por cada e y por cada a y un −1 por cada b y por cada centonces estamos en las mismas condiciones que en el primer problema, pues al borrar2 números los reemplazamos por su producto. Por ejemplo

a → 1

b → −1

c → −1

Entonces podemos saber cuál será el último número, de donde podemos saber enqué par está la última letra entre {e, a} y {b, c}.

De la misma forma, si ponemos un 1 por cada e y cada b y un −1 por cada a ycada c podemos saber en qué par estará la última letra entre {e, b} y {a, c}.

Juntando la información anterior podemos saber cuál será la última letra; alcanzanotar que el primer invariante nos dice en qué columna de la figura que sigue está yel segundo en qué fila.

e ba c

Hemos probado entonces que sin importar lo que hagamos, la última letra siemprees la misma. ‡

La solución anterior se puede reescribir en la siguiente forma más compacta. Reem-placemos cada e por (1, 1), cada a por (1,−1), cada b por (−1, 1) y cada c por (−1,−1).Entonces en cada turno borramos 2 pares y los reemplazamos por su producto (el pro-ducto de 2 pares es coordenada a coordenada). Por ejemplo

a → ( 1,−1)

b → (−1, 1)

c → (−1,−1)

Luego el producto de todos los pares es invariante y este resultado debe ser el parcon el que se termina.

De ahora en adelante por x ∗ y entendemos la letra de la tabla en la fila de “x“ yen la columna de “y“ y lo llamamos su ∗-producto. Por ejemplo, a ∗ b = c, b ∗ b = e ye ∗ a = a.

Por lo recién hecho, no sólo tiene sentido ∗-multiplicar 2 letras sino cualquiercantidad de letras. La ∗-multiplicación de un conjunto de letras es la última letra quequeda luego de aplicar el proceso del problema. Por ejemplo

a ∗ b ∗ a ∗ c = (a ∗ b) ∗ (a ∗ c) = c ∗ b = e

40

Page 41: Invariantes. Todo lo que debías saber para la OMA

Lo anterior tiene sentido porque sin importar en qué orden pongamos los paréntesisy realicemos las operaciones, siempre se llega al mismo resultado. En este contextoel ∗-producto de las letras en el pizarrón es un invariante, pero no podíamos usarlopara resolver el problema porque precisamos del problema para definirlo.

El conjunto anterior con la operación ∗ es el grupo de Klein. En la sección queviene vamos a utilizarlo para resolver algunos problemas pero es importante destacarque no hay nada en especial en éste, todo lo que podemos hacer con el grupo de Kleinlo podemos hacer sin él.

Cada tabla como las anteriores define una operación entre ciertos elementos. Lastablas que vimos en esta sección eran conmutativas, es decir que x ∗ y = y ∗ x paracualquier par de elementos x, y. Sin embargo esto no es necesario como muestra lasiguiente tabla que resume el último problema de la sección anterior.

∗ a b c d e fa a b c d e fb b c a f d ec c a b e f dd d e f a b ce e f d c a bf f d e b c a

Cuando la tabla no es conmutativa ya no tiene sentido ∗-multiplicar conjuntos deletras sino secuencias (finitas) de letras. Veamos un ejemplo usando la tabla anteriorpara ∗-multiplicar b ∗ d ∗ f ∗ e:

b ∗ d ∗ f ∗ e = (b ∗ d) ∗ f ∗ e = f ∗ (f ∗ e) = f ∗ c = e

b ∗ d ∗ f ∗ e = b ∗ (d ∗ f) ∗ e = (b ∗ c) ∗ e = a ∗ e = e

b ∗ d ∗ f ∗ e = b ∗ d ∗ (f ∗ e) = b ∗ (d ∗ c) = b ∗ f = e

Sin importar en qué orden hagamos las operaciones siempre nos va a dar e, poreso es que decimos que b ∗ d ∗ f ∗ e = e. El hecho de que no importe el orden delas operaciones es justamente el problema antes mencionado. De forma análoga paracada tabla podemos tratar de definir una operación. La pregunta es...... ¿Qué tablasconducen a operaciones para las que sin importar el orden en que las realicemossiempre dan lo mismo?

41

Page 42: Invariantes. Todo lo que debías saber para la OMA

Algunos problemas para pensar

1. Tenemos una secuencia de letras, en cada turno podemos elegir 2 letras consec-utivas y cambiarlas por otra de acuerdo a la siguiente tabla:

e a b ce e a b ca a b c db b c e ac c d a e

Demostrar que sin importar lo que hagamos la última letra siempre es la misma.

2. Tenemos una secuencia de letras, en cada turno podemos elegir 2 letras consec-utivas y cambiarlas por otra de acuerdo a una tabla. Sea x ∗ y el elemento en lafila de la “x“ y la columna de la “y“ y supongamos que para todo a, b, c:

(a ∗ b) ∗ c = a ∗ (b ∗ c)

Probar que sin importar lo que hagamos la última letra siempre es la misma.

Nota: Esta propiedad caracteriza a las “buenas tablas“.

3. En el pizarrón están escritos los números 1, 12, 1

3, .., 1

n. En cada turno se deben

elegir 2 de ellos, digamos a y b, borrarlos y escribir en su lugar ab + a + b. ¿Quénúmero queda después de n− 1 turnos?

4. (Rusia 1998) Una operación a ∗ b se define en el conjunto de los números realestal que (a ∗ b) ∗ c = a + b + c probar que a ∗ b = a + b.

5. (Leningrado 1989) Se tiene una operación a ∗ b entre los números enteros (esdecir que a cada par de enteros a, b le asocia otro a ∗ b) tal que todo entero c esde la forma c = a ∗ b para algún par a, b de enteros. Probar que no es posibleque (a ∗ b) ∗ c = a ∗ (b ∗ c) y a ∗ b = −b ∗ a.

42

Page 43: Invariantes. Todo lo que debías saber para la OMA

2.10. Senku

El Senku es un juego del Siglo XV III inventado por un aristócrata francés en suaislamiento en una celda de la Bastilla. Si bien se puede jugar en muchos tableros elobjetivo es siempre el mismo: dejar la menor cantidad posible de fichas.

Se tiene un tablero de 8 × 8 con una ficha en cada casilla salvo en lasesquinas. En cada turno debemos elegir 3 casillas formando un rectángulode 1 × 3 con un extremo vacío y las otras 2 casillas ocupadas. A contin-uación sacamos la ficha de la casilla del medio y a la ficha en un extremola ponemos en el otro extremo.

• • → ••• →

¿Es posible jugar de forma que quede una sola ficha en el tablero?

Pongamos en cada casilla del tablero un elemento del grupo de Klein de la siguientemanera:

a b c a b c a bc a b c a b c ab c a b c a b ca b c a b c a bc a b c a b c ab c a b c a b ca b c a b c a bc a b c a b c a

No es difícil ver que el ∗-producto de los elementos del grupo de Klein en casillasocupadas por fichas se mantiene invariante. Inicialmente el producto es

a20 ∗ b20 ∗ c20 = e ∗ e ∗ e = e

Si logramos dejar una sola ficha, ésta debe estar en una casilla con una e. Comoninguna casilla tiene una e, esto es imposible; entonces no es posible dejar sólo unaficha. ‡

Como dijimos antes, no es necesario usar el grupo de Klein. Usando la mismadistribución de letras que en la demostración anterior, notemos que en cada salto la

43

Page 44: Invariantes. Todo lo que debías saber para la OMA

cantidad de letras a alterna su paridad, pues o borramos una a o agregamos una a.De la misma forma la cantidad de letras b alterna su paridad y la cantidad de letrasc también lo hace. Como inicialmente hay 20 letras de cada una, entonces siempredeben tener la misma paridad, es decir o hay una cantidad par de cada letra o hayuna cantidad impar de cada letra. Si logramos dejar una sola letra, entonces habráuna cantidad impar de una letra y una cantidad par de las otras 2, lo que es absurdo.

Otra alternativa es la siguiente: Consideremos por ejemplo las siguientes distribu-ciones de números

1 1 1 1 1 1 1 −1−1 1 −1 −1 1 −1 −1 1−1 −1 1 −1 −1 1 −1 −1

1 −1 −1 1 −1 −1 1 −1−1 1 −1 −1 1 −1 −1 1−1 −1 1 −1 −1 1 −1 −1

1 −1 −1 1 −1 −1 1 −1−1 1 −1 −1 1 −1 −1 1

−1 1 −1 −1 1 −1 −1 1−1 −1 1 −1 −1 1 −1 −1

1 −1 −1 1 −1 −1 1 −1−1 1 −1 −1 1 −1 −1 1−1 −1 1 −1 −1 1 −1 −1

1 −1 −1 1 −1 −1 1 −1−1 1 −1 −1 1 −1 −1 1−1 −1 1 −1 −1 1 −1 −1

Como cada rectángulo de 1× 3 tiene dos −1 y un 1, en cada distribución el producto delos números en las casillas con fichas debe ser invariante. Inicialmente el producto es 1 enambos, entonces de quedar una sola ficha ésta debe estar en una casilla con un 1 en ambostableros. Pero como se puede ver, no hay ninguna casilla con un 1 en ambos tableros.

La demostración anterior nos muestra como si bien a veces algunos invariantes porseparado no alcanzan, al juntar la información de ambos logramos resolver el problema. Lademostración usando el grupo de Klein no es más que usar 2 invariantes al mismo tiempo.

Algunos problemas para pensar

1. Se tiene un tablero de 4 × 4 y 15 fichas. Se deben ubicar las 15 fichas una en cadacasilla dejando una casilla vacía y luego jugar al Senku. ¿Qué casilla nos convienedejar vacía si queremos dejar sólo 1 ficha al final?

2. (34 Imo 1993) Sobre un tablero de ajedrez infinito se juega de la siguiente manera: Alprincipio hay n2 fichas dispuestas sobre el tablero en un cuadrado de n×n de casillasadyacentes, con una ficha en cada casilla. Cada jugada es un salto de una ficha endirección horizontal o vertical sobre una casilla adyacente, ocupada por otra, hastauna no ocupada, contigua a ella. La ficha sobre la que se ha saltado se retira (comoen el Senku). Halle los valores de n para los que el juego puede terminar quedandouna única ficha en el tablero.

44

Page 45: Invariantes. Todo lo que debías saber para la OMA

3. Se tiene un tablero como el de la figura con una ficha en cada casilla salvo en el centro.Las fichas se comen como en el Senku. ¿Es posible dejar 1 sola ficha y que ésta quedeen la casilla marcada? ¿Y dejar una sola en el centro?

4. Supongamos que tenemos un grafo plano en donde todos los vértices tienen 3 aristas.Probar que es posible pintar las aristas con 3 colores, de forma tal que no haya dos delmismo color que tengan un vértice en común, si y sólo si se pueden pintar las carascon 4 colores de forma que no haya 2 caras con un lado en común del mismo color.

45

Page 46: Invariantes. Todo lo que debías saber para la OMA

2.11. RanasEl siguiente juego lo propuso J.H. Conway. Se mueve como en el Senku aunque también

tiene un aire al problema “Piedras“.

En un tablero infinito hay una línea que llamaremos horizonte. En algunascasillas por debajo del horizonte hay ranas, a lo sumo una por casilla. Lasranas se mueven de la siguiente manera, se pueden mover en horizontal o envertical saltando por encima de otra rana contigua siempre que la casilla a laque salte esté vacía, comiendo la rana sobre la que saltó. ¿Es posible para algunadisposición inicial de las ranas que una de ellas llegue a la quinta fila por encimadel horizonte?

Supongamos que tenemos una disposición inicial para las ranas, totalmente contenidapor debajo del horizonte, que les permita llegar a una casilla en la quinta fila (a la quellamaremos “casilla especial“). Podemos suponer que la cantidad de ranas es finita, pues lasinvolucradas para que una de ellas alcance la quinta fila debe serlo.

Sea w =√

5−12 , que es la raíz positiva de la ecuación x2 +x = 1. Notemos que 0 < w < 1

y escribamos en cada casilla una potencia de w de la siguiente forma: pongamos un 1 en lacasilla especial y en las demás casillas potencias de w como muestra la figura.

1w2 w w2

w4 w3 w2 w3 w4

w6 w5 w4 w3 w4 w5 w6

w8 w7 w6 w5 w4 w5 w6 w7 w8

w9 w8 w7 w6 w5 w6 w7 w8 w9

w9 w8 w7 w6 w7 w8 w9

w9 w8 w7 w8 w9

w9 w8 w9

46

Page 47: Invariantes. Todo lo que debías saber para la OMA

El peso de una rana es el número en la casilla que ocupa y naturalmente el peso totales la suma de los pesos de todas las ranas. La situación ideal sería que en cada salto elpeso total se mantenga invariante. Si bien esto no es cierto, lo que podemos ver es que estacantidad nunca crece.

En efecto, supongamos que la rana sobre la que se salta tenía peso wn+1 y que no estabaen la misma columna que la casilla especial. Entonces la rana que saltó tenía peso wn+2 ypasó a tener peso wn o tenía peso wn y pasó a tener peso wn+2. En ambos casos el peso totalno crece, pues la rana sobre la que se saltó se retira y como w+w2 = 1 → wn+1+wn+2 = wn

entonces:

1. wn+1 + wn+2 ≥ wn

2. wn + wn+1 ≥ wn+2

Falta analizar qué pasa si se saltó sobre una rana en la columna de la casilla especial.Si el salto fue vertical, el mismo argumento que antes funciona; si el salto fue horizontal esmás simple pues la rana que saltó seguirá teniendo el mismo peso pero la otra se retira.

En conclusión, el peso total nunca crece.Ahora es momento de estimar el peso total inicial, para ello debemos sumar todos los

números debajo del horizonte. Vamos a usar12 que w3 + 2w4 + 2w5 + ... = 1 pues

w3 + 2w4 + 2w5 + .... = w3 + w4 + w4 + w5 + w5 + w6 + ....

w3 + 2w4 + 2w5 + .... = w2 + w3 + w4 + ... = w2 11− w

= 1

Luego

1. Los números en la primer fila suman w5 + 2w6 + 2w7 + ... = w2

2. Los números en la segunda fila suman w6 + 2w7 + 2w8 + ... = w3

3. Los números en la tercer fila suman w7 + 2w8 + 2w9 + ... = w4

4. Los números en la cuarta fila suman ........

Entonces todos los números suman w2 + w3 + w4 + .... = w2 11−w = 1. De donde, ini-

cialmente el peso total de las ranas era < 1 (para ser 1 necesitaríamos de infinitas ranas) loque es absurdo pues cuando una rana llegó a la casilla especial ella sola superó el peso total.Hemos probado entonces que no existe ninguna disposición inicial para las ranas por debajodel horizonte que les permita llegar a la quinta fila. ‡

Algunos problemas para pensar

1. Demostrar que el mínimo número de ranas para llegar a la tercer fila sobre el horizontees 8 y para llegar a la cuarta 20.

12Con respecto a las “Sumas infinitas“ consultar el apéndice

47

Page 48: Invariantes. Todo lo que debías saber para la OMA

2.12. MonovariantesSiguiendo el ejemplo de la sección anterior, veamos algunos casos en donde en vez de

usar algo que se mantiene cuando modificamos la situación, usamos algo que siempre varíaen forma monótona (es decir siempre crece o siempre decrece).

En una fiesta hay 600 personas y 10 habitaciones, en cada segundo una personase cambia de habitación de forma que pasa a una que tenía más personas queen la que estaba. Probar que tarde o temprano todas las personas estarán en lamisma habitación.

Supongamos que una habitación tenía m personas y una de ellas se fue a otra que tenían, con n ≥ m. Entonces (n + 1)2 + (m − 1)2 = n2 + m2 + 2(n − m + 1) ≥ n2 + m2 + 2,de donde la suma de los cuadrados de la cantidad de personas en cada habitación siemprecrece estrictamente. Pero si llamamos S a este número, S < 10× 6002, entonces a partir decierto momento, S deja de crecer. Entonces las personas dejan de cambiar de habitación dedonde todas están en la misma. ‡

Dado un polígono no convexo le aplicamos la siguiente operación: elegimos 2vértices no consecutivos A y B, tales que el polígono está contenido en unode los 2 semiplanos que determina la recta AB y se refleja una de las partesdel polígono que une A con B por el punto medio de AB. Si aplicamos estaoperación indefinidamente, probar que tarde o temprano el polígono se vuelveconvexo.

En primer lugar, cada vez que aplicamos la operación al polígono no modificamos suslados ni sus direcciones, solamente cambiamos el orden; luego los posibles polígonos quepodemos obtener son finitos.

48

Page 49: Invariantes. Todo lo que debías saber para la OMA

Por otro lado, el área del polígono siempre crece. Como los posibles polígonos son finitosentonces en algún momento tenemos que dejar de modificarlo pues como el área crece estosdeben ser todos distintos. Pero si dejamos de modificarlo quiere decir que dejamos de aplicarlela operación o lo que es lo mismo que obtuvimos un polígono convexo. ‡

Si en vez de reflejar por el punto medio de AB lo hacemos por la recta AB entoncesel problema sigue valiendo pero es más difícil. Se conjetura que si el polígono inicial tienen lados entonces 2n reflexiones como las anteriores alcanzan para transformarlo en unoconvexo.

Supongamos que tenemos una secuencia de números enteros no todos igualesS = (a, b, c, d). En cada turno los reemplazamos por (a− b, b− c, c− d, d− a).Probar que para todo N , el módulo de alguna coordenada se volverá más grandeque N .

Sea Sn = (an, bn, cn, dn) la secuencia que se obtiene luego de n turnos. Usando quean + bn + cn + dn = 0 si n ≥ 1 se puede ver que

a2n+1 + b2

n+1 + c2n+1 + d2

n+1 = 2(a2n + b2

n + c2n + d2

n) + (an + cn)2 + (bn + dn)2

Entonces a2n+1 + b2

n+1 + c2n+1 + d2

n+1 ≥ 2(a2n + b2

n + c2n + d2

n) de donde

a2n+1 + b2

n+1 + c2n+1 + d2

n+1 ≥ 2n(a21 + b2

1 + c21 + d2

1)

Ahora si tomamos n tal que 2n(a21 + b2

1 + c21 + d2

1) > 4N2 entonces al menos uno entre(an+1, bn+1, cn+1, dn+1) debe tener módulo más grande que N . ‡

Supongamos que tenemos una secuencia de números enteros S = (a, b, c, d). Encada turno los reemplazamos por (|a − b|, |b − c|, |c − d|, |d − a|). Probar quetarde o temprano llegamos a (0, 0, 0, 0).

Sea Sn = (an, bn, cn, dn) la secuencia que se obtiene luego de n turnos y pongamosMi = max(ai, bi, ci, di). Notemos que Mi+1 ≤ Mi con igualdad si y sólo si el mayor de todostiene un vecino que es 0. No es difícil ver que si Mi > 0 entonces Mi+4 < Mi. Como Mn ≥ 0para todo n entonces la secuencia {Mn} debe decrecer hasta llegar a 0. Pero si Mn = 0entonces (an, bn, cn, dn) = (0, 0, 0, 0) como queríamos. ‡

49

Page 50: Invariantes. Todo lo que debías saber para la OMA

Se tiene un tablero infinito con n casillas pintadas de negro. En cada turno sepuede elegir una casilla y si tiene al menos 2 de sus 4 vecinas del color opuestole cambiamos el color. Supongamos que luego de varios turnos hay 2006 casillaspintadas de negro de forma que ningún par de ellas son vecinas y el resto estánde blanco. Probar que n ≥ 2006.

Sea P el perímetro de las casillas negras, esto es la cantidad de lados de casillas negrasque comparten con alguna casilla blanca (dicho de otra forma, P es la cantidad de pares decasillas vecinas de distinto color). En cada turno P se mantiene igual o se hace más chico.Pues por ejemplo, si pintamos una casilla blanca de negro le estamos quitando al menos 2lados al perímetro de las negras y a lo sumo le estamos agregando 2 nuevos (recordar queuna casilla blanca se puede pintar de negro sólo si tenía al menos 2 vecinas negras).

Inicialmente P es a lo sumo 4n. En la situación final el perímetro es 4 × 2006. Luego4n ≥ 4× 2006 es decir que n ≥ 2006. ‡

Algunos problemas para pensar1. (San Petersburgo 1996) Hay varios números enteros escritos en el pizarrón, en cada

turno se deben elegir 2 de ellos, digamos a y b y se los reemplaza por (a, b) y [a, b].Demostrar que luego de cierta cantidad de turnos los números dejarán de cambiar.

2. (MOP 1998) Se tiene un 2000 − agono regular y 2001 semillas distribuidas entre susvértices. En cada turno se debe elegir un vértice con al menos 2 semillas, quitarlas yponer una de ellas en un vértice vecino y la otra en el otro vértice vecino. Probar quetarde o temprano habrá al menos 1001 vértices sin semillas.

3. Probar que los vértices de cualquier grafo se pueden pintar de blanco y negro de formaque para cada vértice, al menos la mitad de sus vecinos tiene el color opuesto.

4. (Irán TST 2005) Se tiene una cantidad finita de puntos en el plano y un círculo. Encada turno se calcula el baricentro de los puntos en el interior del círculo y se muevesu centro a este punto. Probar que luego de varios turnos el círculo va a dejar demoverse.

5. (Rumania TST 2002) Todo miembro del parlamento tiene su rating personal. El par-lamento se divide en grupos y cada miembro tiene un rating relativo, que es el cocientede su rating personal sobre la suma de los ratings personales de todos los miembrosdel grupo. Un miembro del parlamento se puede cambiar de un grupo a otro sólo sien el nuevo grupo su rating relativo aumenta. Cada día, solamente un miembro delparlamento puede cambiar de grupo. Demostrar que sólo son posibles un número finitode movimientos.

6. Se tiene un tablero de 10× 10 con algunas casillas infectadas. En cada segundo cadacasilla con al menos 2 vecinas infectadas se infecta (2 casillas son vecinas si tienen unlado en común). Si inicialmente había 9 casillas infectadas, ¿es posible que la infecciónllegue a todo el tablero?

50

Page 51: Invariantes. Todo lo que debías saber para la OMA

2.13. Un problema y 2 solucionesEl tercer problema de la 27 Imo de 1986 decía así:

En cada vértice de un pentágono hay escrito un número entero de forma quela suma de todos es positiva. Si x, y, z están en vértices consecutivos con y < 0entonces los podemos cambiar por x + y,−y, y + z. La operación anterior serepite siempre que se pueda. Probar que en algún momento todos los númerosse van a volver positivos.

Sea s la suma de los 5 números escritos, s > 0 y como x + y + z = x + y − y + y + zentonces s se mantiene invariante con cada operación.

Para cada pentágono con los números x1, x2, x3, x4, x5 en sus vértices vamos a decir quesu “valor“ es

(x1 − x3)2 + (x3 − x5)2 + (x5 − x2)2 + (x2 − x4)2 + (x4 − x1)2

Si vn es el valor del pentágono después de n operaciones entonces vn+1 < vn. Pues si porejemplo el cambio fue (x2, x3, x4) → (x2 + x3,−x3, x3 + x4) (los demás casos son análogos)entonces

vn = (x1 − x3)2 + (x3 − x5)2 + (x5 − x2)2 + (x2 − x4)2 + (x4 − x1)2

vn+1 = (x1−(−x3))2+(−x3−x5)2+(x5−(x2+x3))2+((x2+x3)−(x4+x3))2+((x3+x4)−x1)2

Luego vn+1 − vn = 2sx3 < 0 (cuestión de hacer la cuenta). Ahora es fácil terminar elproblema, alcanza notar que la sucesión de enteros no negativos {vn} decrece indefinida-mente, lo que es imposible. Entonces debe ser finita, es decir que sólo se pueden realizar unacantidad finita de operaciones o lo que es lo mismo que después de una cantidad finita depasos el pentágono tendrá todos los vértices con números positivos. ‡

La solución anterior, si bien interesante y muy ingeniosa, no nos dice nada acerca dequé ocurre en el caso en que los números no sean necesariamente enteros y nos da pocainformación acerca de cuántas operaciones hay que realizar o cuál es la posición final. Veamosotra solución que se acerca más a estas preguntas.

Pongamos x1 = x6 = x11 = ... , x2 = x7 = ... , ...., x5 = x10 = ... y consideremos elconjunto infinito T de sumas de la forma xi + xi+1 + ... + xj con 1 ≤ i ≤ 5 y i ≤ j, porejemplo

x3 + x4 + x5 + x6 + x7 + x8 + x9 + x10 + x11

En cada cambio del tipo (x2, x3, x4) → (x2 +x3,−x3, x3 +x4) el conjunto T se mantieneigual salvo que x3 cambia por −x3. Lo que pasa es que cada suma se mantiene o se inter-cambia con otra, por ejemplo

x3 + x4 + x5 → (−x3) + (x3 + x4) + x5 = x4 + x5

51

Page 52: Invariantes. Todo lo que debías saber para la OMA

x4 + x5 → (x3 + x4) + x5 = x3 + x4 + x5

Es decir que en cada paso un elemento negativo del conjunto T se cambia a positivoy el resto queda igual. Si bien el conjunto es infinito, como s > 0, solamente puede teneruna cantidad finita de elementos negativos. Entonces a lo sumo podemos realizar finitasoperaciones, es decir que después de una cantidad finita de cambios todos los números seránpositivos.

Algunos problemas para pensar

1. Probar que en el problema anterior, no importa qué operaciones hagamos, siempre setarda la misma cantidad de pasos en lograr que todos sean positivos.

2. (Olimpíada de Leningrado 1990) Son dados 20 números en un círculo. Si x, y, z sonnúmeros consecutivos, se los puede cambiar por x + y,−y, y + z. ¿Es posible cam-biar la 20−upla (1, 2, 3, 4, 5, 6, 7, 8, 9, 10,−1,−2,−3,−4,−5,−6,−7,−8,−9,−10) por(10, 9, 8, 7, 6, 5, 4, 3, 2, 1,−10,−9,−8,−7,−6,−5,−4,−3,−2,−1) usando estas opera-ciones?

3. (Olimpíada de Leningrado 1990) Son dados varios números en un círculo. Si x, y, z sonnúmeros consecutivos, se los puede cambiar por x + y,−y, y + z. Probar que luego deuna cantidad finita de estas operaciones se puede conseguir que todos sean positivosy demostrar que la posición final es única.

52

Page 53: Invariantes. Todo lo que debías saber para la OMA

2.14. Diamante AztecaSupongamos que tenemos un tablero de 2n × 2n cuyas filas están numeradas de abajo

hacia arriba −n, ..,−1, 1, ..., n y sus columnas de izquierda a derecha −n, ...,−1, 1, .., n (notarque no hay ni fila ni columna 0). La casilla (x, y) es la que está en la columna x y la filay. El Diamante Azteca de orden n es la figura formada por las casillas (x, y) tales que|x|+ |y| ≤ n + 1.

Cada vez que digamos que el Diamante está cubierto por fichas de dominó (cada una delas cuales cubre 2 casillas que comparten un lado) estamos asumiendo implícitamente quelas fichas no se superponen ni salen fuera del Diamante.

Figura 1: Diamante Azteca de orden 4 y rotaciones permitidas

Se tiene un Diamante Azteca de orden n cubierto por fichas de dominó verticales.En cada turno podemos rotar 90o cualquier cuadrado de 2 × 2 cubierto porexactamente 2 dominós. Probar que la menor cantidad de rotaciones necesariaspara lograr que todos los dominós queden horizontales es n(n+1)(2n+1)

6 .

En primer lugar veamos como ir de “todos verticales“ a “todos horizontales“ y tratemosde contar cuántas rotaciones usamos. Nos será de gran ayuda la siguiente observación:

“Supongamos que tenemos 2 dominós verticales a la misma altura separados por 2k − 2columnas. Si el espacio entre ellos está completamente cubierto por dominós horizontalesentonces con 2k − 1 rotaciones podemos lograr que los 2k dominós estén horizontales.“

2k Columnas

53

Page 54: Invariantes. Todo lo que debías saber para la OMA

Para ir de “todos verticales“ a “todos horizontales“ la estrategia es la siguiente: en la primeretapa nos ocupamos de poner los dominós en las columnas −1 y 1 horizontales, en la segundatenemos que lograr que las columnas −2,−1, 1, 2 queden cubiertas por dominós horizontalesy el resto por dominós verticales, .... , en la k−ésima etapa tenemos que lograr que lascolumnas −k, ...., k queden cubiertas por dominós horizontales y el resto por verticales.Después de la n−ésima etapa todas las columnas estarán cubiertas por dominós horizontalesy habremos conseguido llegar a “todos horizontales“. Veamos que esto es posible y calculemoscuántas rotaciones usamos.

Figura 2: Etapa 1 y par para la Etapa 2

Es fácil ver que en la primer etapa usamos solamente n rotaciones. En la segunda etapa,asociemos a cada dominó vertical en la columna −2 otro en la columna 2 a la mismaaltura (por ejemplo, los dominós marcados en la figura 2). Como el espacio entre ellosestá completamente cubierto por dominós horizontales entonces, por la observación, en 3rotaciones podemos hacer que estén todos horizontales. Como hay n−1 pares, uno por cadadominó vertical en la columna −2, entonces en esta etapa usamos 3(n− 1) rotaciones.

Figura 3: Etapa 3 y pares de dominós a la misma altura asociados

54

Page 55: Invariantes. Todo lo que debías saber para la OMA

Al principio de la k−ésima etapa tenemos las columnas −(k− 1), ...., k− 1 cubiertas pordominós horizontales y el resto por dominós verticales. Hay n− k + 1 dominós verticales enla columna −k, a cada uno de ellos lo asociamos a otro dominó vertical a la misma altura enla columna k (como en la figura 3). Como entre ellos hay 2k − 2 columnas completamentecubiertas por dominós horizontales entonces en 2k − 1 rotaciones podemos hacer que esténtodos horizontales. Como hay n − k + 1 pares precisamos (2k − 1)(n − k + 1) rotaciones.Notemos que al finalizar esta etapa, las columnas −k, ..., k están completamente cubiertaspor dominós horizontales y el resto por dominós verticales.

Para saber cuantas rotaciones usamos en total debemos sumar (2k−1)(n−k+1) para kentre 1 y n ya que después de n etapas habremos llegado a “todos horizontales“. La cantidadde rotaciones usadas es13

n∑k=1

(2k − 1)(n− k + 1) =n(n + 1)(2n + 1)

6

Veamos que en efecto la cantidad anterior es la menor posible. Para ello escribamos encada casilla un número como muestra la figura. El rango de un cubrimiento del DiamanteAzteca por dominós es la suma de los números en las casillas inferior izquierda de cadadominó. ¿Qué pasa con el rango después de cada rotación?

Figura 4: El cubrimiento de la derecha tiene rango 16

En cada rotación, el rango crece o disminuye en 1. Para probar lo anterior alcanza conanalizar la figura 5. Supongamos que los dominós que rotamos tenían en su casilla inferiorizquierda uno el número a y el otro b y pasaron a tener uno el número b y el otro c. Como lascasillas en una misma diagonal que comparten un vértice tienen números consecutivos, esdecir |a− c| = 1, entonces el rango creció en 1 o disminuyó en 1 (dependiendo de si a = c+1o a = c− 1).

13Consultar el apéndice “Sumas de Potencias“

55

Page 56: Invariantes. Todo lo que debías saber para la OMA

Figura 5: Variación del rango en una rotación

Es fácil ver que el rango de “todos horizontales“ es 0 pues cada ficha aporta lo opuestoque la ficha simétrica con respecto al eje de simetría horizontal del Diamante. Por otro lado,calcular el rango de “todos verticales“ es ligeramente mas complicado. En la mitad izquierdadel Diamante la columna −k aporta −(n−k+1)2 al rango y en la mitad derecha la columnak aporta (n− k + 1)(n + k). Luego el rango de “todos verticales“ es14:

n∑k=1

(n− k + 1)(n + k)−n∑

k=1

(n− k + 1)2 =n∑

k=1

(n− k + 1)(2k − 1)

n∑k=1

(n− k + 1)(n + k)−n∑

k=1

(n− k + 1)2 =n(n + 1)(2n + 1)

6

Como con cada rotación el rango a lo sumo disminuye en 1, para ir de “todos verticales“a “todos horizontales“ precisamos al menos n(n+1)(2n+1)

6 rotaciones. ‡

No es difícil probar que para todo cubrimiento T de rango r(T ) valen las desigualdades:

0 ≤ r(T ) ≤ n(n + 1)(2n + 1)6

Más aun, el único cubrimiento de rango nulo es “todos horizontales“ y el único cubrim-iento de rango máximo es “todos verticales“. Por otro lado, r(T ) no apareció en la soluciónanterior por casualidad. Se puede probar que la menor cantidad de rotaciones para ir deun cubrimiento T hasta “todos horizontales“ es r(T ), su rango. Esto es remarcable, pues enprincipio ni siquiera sabemos que sea cierto que se puede llegar en un número arbitrario derotaciones.

Notemos que la noción de “rango“ de un cubrimiento no sólo nos dice cuál es la menorcantidad de rotaciones necesarias para que todos queden horizontales sino que nos propor-ciona de una forma para hacerlo, alcanza con siempre elegir para rotar un cuadrado quehaga bajar el rango en 1.

Otro problema relacionado al Diamante Azteca es el de determinar, para cada n, lacantidad de cubrimientos por fichas de dominó del Diamante Azteca de orden n. Podemos

14Consultar el apéndice antes mencionado

56

Page 57: Invariantes. Todo lo que debías saber para la OMA

ir más lejos todavía y preguntarnos: ¿Será posible calcular, para cada r y cada n, cuántoscubrimientos del Diamante Azteca de orden n hay de rango r?

Antes de dar una respuesta a la pregunta anterior es preciso hacer una pequeña obser-vación. En “todos horizontales“ no hay ningún dominó vertical y en cada rotación o paramos2 dominó o acostamos 2 dominó, luego la cantidad de dominós verticales debe ser siemprepar. Como desde “todos horizontales“ podemos llegar a cualquier cubrimiento por rotacionesentonces para cualquier cubrimiento la cantidad de dominós verticales es par.

Dado un cubrimiento T , por r(T ) denotamos su rango y por v(T ) la mitad de la cantidadde dominós verticales en T . En la siguiente suma, T recorre todos los cubrimientos delDiamante Azteca de orden n por dominós.

Pn(x, y) =∑T

xv(T )yr(T )

El polinomio Pn(x, y) no es más que una forma de guardar información. El coeficientede xvyr de Pn(x, y) es la cantidad de cubrimientos con 2v dominós verticales y rango r. Larazón para guardar esta información de esta extraña manera es que Pn(x, y) es más fácil derecordar que cada uno de sus coeficientes pues15:

Pn(x, y) =n−1∏k=0

(1 + xy2k+1)n−k

Veamos como usar la formula anterior. Si por ejemplo deseamos saber cuántos cubrim-ientos del Diamante Azteca de orden n hay, debemos poner x = y = 1 pues de esa formacada cubrimiento contribuye con 1 en la formula de Pn(x, y), luego

Pn(1, 1) =n−1∏k=0

(1 + 12k+1)n−k =n−1∏k=0

2n−k = 2n(n+1)

2

Es decir que hay 2n(n+1)

2 cubrimientos. Si ponemos x = 1, entonces cada cubrimientode rango r contribuye con yr a Pn(1, y), luego la cantidad de cubrimientos de rango r es elcoeficiente de yr de Pn(1, y).

Pn(1, y) =n−1∏k=0

(1 + y2k+1)n−k

15En realidad es más sutil; si por ejemplo queremos saber cuántos cubrimientos hay, alcanza conevaluar el polinomio en un valor dado, pero si sólo tuviéramos la lista de sus coeficientes, sumarlospodría resultar demasiado complicado. Por otro lado, a veces es más fácil obtener la expresión delpolinomio que cada uno de sus coeficientes por separado. El último problema es un ejemplo de estasituación.

57

Page 58: Invariantes. Todo lo que debías saber para la OMA

Por último, si ponemos y = 1 entonces cada cubrimiento con 2v dominós verticalescontribuye con xv a Pn(x, 1).

Pn(x, 1) =n−1∏k=0

(1 + x)n−k = (1 + x)n(n+1)

2

Algunos problemas para pensar

1. (San Petersburgo 1997) Se tiene el Diamante Azteca de orden n cubierto por fichasde dominó. Probar que en a lo sumo n(n+1)(2n+1)

6 rotaciones podemos conseguir quetodos los dominós estén horizontales.

2. Se tiene un tablero con m filas y n columnas, con n par, cubierto por fichas dedominó. Probar que mediante “rotaciones“ se puede hacer que todas las fichas quedenhorizontales.

3. Demostrar que hay exactamente 2n(n+1)

2 formas de cubrir al Diamante Azteca de ordenn con fichas de dominó.

4. Probar que para todo cubrimiento T de rango r(T ) de un Diamante Azteca de ordenn vale que:

0 ≤ r(T ) ≤ n(n + 1)(2n + 1)6

Hallar los casos donde se da la igualdad.

5. Probar que desde cualquier cubrimiento de un Diamante Azteca se puede hacer unarotación que baje en 1 el rango. Usar esto y el problema anterior para dar otra de-mostración del problema 1.

6. Supongamos que tenemos n puntos numerados 1, 2, 3, ..., n. Un árbol T es una colecciónde n − 1 segmentos entre los n puntos de forma que desde cualquier punto se puedallegar a cualquier otro viajando por los segmentos. En la siguiente suma, T recorretodos los árboles en estos n puntos y para cada uno de ellos di(T ) es la cantidad desegmentos que usan al punto i de vértice.

P (x1, ...., xn) =∑T

xd1(T )−11 x

d2(T )−12 ... xdn(T )−1

n

Probar que P (x1, ..., xn) = (x1 + ... + xn)n−2. En particular hay nn−2 árboles en npuntos numerados. ¿Cuántos hay con d1, d2, ..., dn segmentos con vértices en 1, 2, .., nrespectivamente?

58

Page 59: Invariantes. Todo lo que debías saber para la OMA

2.15. PulgasPara terminar, un hermoso problema de la 41 Imo del 2000. El problema es realmente

bueno.

Sea n ≥ 2 un número entero positivo. Inicialmente hay n pulgas en una rectahorizontal y no todas están en el mismo punto. Para un número real positivo λ,definimos un salto como sigue:

Se eligen 2 pulgas situadas en puntos A y B con A a la izquierda de B. Luegola pulga situada en A salta hasta el punto C de la recta, a la derecha de B, talque BC

AB = λ.

Determinar todos los valores de λ tales que para cualquier punto M de la rectay cualesquiera posiciones iniciales de las n pulgas, existe una sucesión finita desaltos que permite situar a todas las pulgas a la derecha de M .

En primer lugar notemos que alcanza con situar una sola pulga a la derecha de M yaque después hacemos saltar al resto sobre esta pulga y logramos poner todas a la derechade M .

La primer pulga es la que está más a la derecha y la última es la que está más hacia laizquierda (los nombres de las pulgas pueden cambiar después de cada salto). Si tenemos 2posiciones para n pulgas, digamos en los puntos p1, ..., pn la primera y p∗1, ..., p

∗n la segunda

con pn y p∗n las pulgas más a la derecha respectivamente, vamos a decir que la segundaposición avanzó d con respecto a la primera si p∗n está a la derecha de pn y pnp∗n = d.

Definamos el combustible de un conjunto de pulgas como la suma de las distancias dela primer pulga a las otras n − 1 pulgas. A continuación vamos a analizar qué pasa conel combustible después de cada salto. Supongamos entonces que antes del i−ésimo salto elcombustible es Ci y después de éste las pulgas avanzan di.

Si la primera pulga sigue siendo la misma, es decir di = 0, es claro que el combustibledisminuye:

Ci − Ci+1 > 0 (1)

En caso contrario, supongamos que una pulga en A salta hasta C superando a laprimer pulga que está en B. Entonces di = BC y BC

BA ≤ λ (con igualdad si y sólo si lapulga saltó sobre la primera). Entonces

Ci − Ci+1 = BA−BC − (n− 2)BC

59

Page 60: Invariantes. Todo lo que debías saber para la OMA

Ci − Ci+1 = BC(BA

BC− (n− 1))

Ci − Ci+1 ≥ di(1λ− (n− 1)) (2)

Nos dividimos en 2 casos

1. Si λ < 1n−1 entonces λ∗ = 1

λ − (n − 1) > 0. La idea es entonces que para avanzar ddeben gastar al menos dλ∗ combustible y como éste es finito no pueden avanzar tantocomo quieran. Para ser más precisos, (1) y (2) se resumen en

Ci − Ci+1 ≥ diλ∗

Pues en (1) podemos tomar di = 0, luego

C1 = C1 − C2 + C2 − C3 + ... + Ci − Ci+1 + Ci+1

C1 ≥ d1λ∗ + d2λ

∗ + ... + diλ∗ + Ci+1

C1 ≥ (d1 + d2 + ... + di)λ∗

O lo que es lo mismo:

d1 + .... + di ≤C1

λ∗

Pero después de los primeros “i“ saltos las pulgas avanzaron, en total, d1 +d2 + ...+di.Es decir que las pulgas nunca podrán avanzar más que C1

λ∗ .

2. Si λ ≥ 1n−1 entonces hacemos que en el i−ésimo salto la última pulga salte sobre la

primera (esto garantiza que se da la igualdad en (2)). Luego Ci ≤ Ci+1 ya que

Ci − Ci+1 = di(1λ− (n− 1)) ≤ 0

Por otro lado di ≥ Cin−1λ, pues antes del i−ésimo salto la primer pulga y la última

estaban a distancia ≥ Cin−1 (por la definición de Ci). Luego di ≥ Ci

n−1λ ≥ C1n−1λ.

Como di ≥ C1n−1λ para todo “i“ entonces las pulgas avanzan algo fijo en cada salto

entonces tarde o temprano superarán cualquier punto M de la recta.

Hemos probado entonces que las pulgas pueden ubicarse todas a la derecha de cualquierpunto M de la recta si y sólo si λ ≥ 1

n−1 . ‡

60

Page 61: Invariantes. Todo lo que debías saber para la OMA

3. Apéndices

61

Page 62: Invariantes. Todo lo que debías saber para la OMA

3.1. Sumas infinitas“Los ignorantes suponen que infinitos sorteos requieren un tiempo infinito; en realidad

basta que el tiempo sea infinitamente subdivisible, como lo enseña la famosa parábola delCertamen con la Tortuga“ J.L.Borges: La Lotería de Babilonia

Pensemos en Aquiles y la Tortuga. Luego de que Aquiles trate sin éxito de atraparla,ésta se apiada del guerrero y decide esperarlo. Desde el momento en que la Tortuga dejade escapar, Aquiles debe primero alcanzar el punto medio del camino que los separa. Unavez hecho esto, debe alcanzar el punto medio de la segunda mitad del camino y luego elpunto medio de la segunda mitad de la segunda mitad del camino y así indefinidamente. Siel camino medía 2, y como sabemos que Aquiles alcanza la tortuga, podemos decir que

1 +12

+14

+ .... +12n

+ .... = 2 (1)

Pero..... ¿Qué es lo que justifica la igualdad anterior? Después de todo, sólo sabemossumar finitos números. Lo que podemos hacer es ir calculando las “sumas parciales“ y tratarde ver a qué se parecen.

S0 = 1 = 1

S1 = 1 +12

= 1, 5

S2 = 1 +12

+14

= 1, 75

S3 = 1 +12

+14

+18

= 1, 875

S4 = 1 +12

+14

+18

+116

= 1, 9375

Como se observa, las sumas parciales se van acercando cada vez más a 2. En efecto, comopara pasar de Sn a Sn+1 sumamos la mitad de lo que nos falta para llegar a 2, entonces

|2− Sn+1| =12|2− Sn|

Esto justifica que las sumas parciales están cada vez más cerca de 2, tan cerca como quer-amos, y de allí que (1) tiene sentido.

Ahora tratemos de generalizar lo anterior y para x tal que |x| < 1 vamos a calcular

1 + x + x2 + .... + xn + ....

Lo primero que haremos es buscar una fórmula para las sumas parciales, esto es 1+x+...+xn

x(xn + .... + x + 1) = xn+1 + xn + ... + x

62

Page 63: Invariantes. Todo lo que debías saber para la OMA

x(xn + .... + x + 1) = xn+1 + xn + ... + x + 1− 1

x(xn + .... + x + 1) = xn+1 − 1 + xn + ... + x + 1

(x− 1)(xn + .... + x + 1) = xn+1 − 1

xn + .... + x + 1 =xn+1 − 1

x− 1O lo que es lo mismo

xn + .... + x + 1 =1− xn+1

1− x(2)

Ahora analicemos qué pasa con (2) cuando n se va haciendo más y más grande. Como|x| < 1, entonces xn+1 se hace cada vez más chico (tanto como queramos). Luego paravalores grandes de n, el numerador en (2) es muy parecido a 1 y como el denominador essiempre el mismo, podemos decir que las sumas parciales son cada vez más parecidas a 1

1−x .Pongamos entonces

1 + x + x2 + .... + xn + .... =1

1− x

Recordemos que la fórmula anterior solamente vale para |x| < 1. Sólo para asegurarnosque le dimos el valor correcto, calculemos que tan cerca está la n−ésima suma parcialSn = 1 + x + ... + xn de S = 1

1−x :

S − Sn =1

1− x− (1 + x + ... + xn)

S − Sn =1

1− x− 1− xn+1

1− x

S − Sn =xn+1

1− x

Como |x| < 1, entonces S−Sn se va haciendo cada vez más chico. Resumiendo, repitamoslas 2 fórmulas que hemos visto:

1 + x + .... + xn =1− xn+1

1− x

y si |x| < 1 entonces

1 + x + .... + xn + .... =1

1− x

Por ejemplo, si x = 12 entonces

1 +12

+14

+ .... +12n

+ ... =1

1− 12

63

Page 64: Invariantes. Todo lo que debías saber para la OMA

1 +12

+14

+ .... +12n

+ ... = 2

Si x = 13 entonces:

1 +13

+19

+ .... +13n

+ ... =1

1− 13

1 +13

+19

+ .... +13n

+ ... =32

Por último si w es tal que w2 + w = 1 entonces

1 + w + w2 + ... + wn + ... =1

1− w

1 + w + w2 + ... + wn + ... =1

w2

64

Page 65: Invariantes. Todo lo que debías saber para la OMA

3.2. Sumas de potenciasEs conocida la historia de cómo Gauss sumó los números del 1 al 100 con una simple

cuenta. Lo primero que hizo fue escribirlos en una fila de menor a mayor. Después los volvióa escribir en fila, debajo de la primera, pero esta vez de mayor a menor.

1 2 3 ..... 99 100100 99 .. ..... 2 1

En cada columna los 2 números suman 101, hay 100 columnas de donde los númerosdeben sumar 100×101. Como cada número está escrito dos veces, la suma deseada es 100×101

2 .De forma más general, con la misma idea podemos ver que:

1 + 2 + .... + n =n× (n + 1)

2(1)

Otra manera de probar (1) es por inducción. La fórmula es cierta para n = 1, ahorasupongámosla cierta para n = k y veámoslo para n = k + 1.

1 + 2 + ... + (k + 1) = (1 + 2 + ... + k) + k + 1

1 + 2 + ... + (k + 1) =k(k + 1)

2+ k + 1

1 + 2 + ... + (k + 1) =k(k + 1) + 2(k + 1)

2

1 + 2 + ... + (k + 1) =(k + 1)(k + 2)

2Como la fórmula vale para n = 1, por lo que acabamos de probar también debe valer

para n = 2 y entonces también lo hace para n = 3 y así vemos que vale para todo n.De la misma forma que acabamos de probar (1) podemos demostrar:

12 + 22 + ... + k2 =k(k + 1)(2k + 1)

6(2)

En efecto, la fórmula vale para n = 1, supongámosla cierta para n = k y veámoslo paran = k + 1

12 + 22 + ... + (k + 1)2 = (12 + 22 + ... + k2) + (k + 1)2

12 + 22 + ... + (k + 1)2 =k(k + 1)(2k + 1)

6+ (k + 1)2

12 + 22 + ... + (k + 1)2 =(k + 1)(k(2k + 1) + 6(k + 1))

6

12 + 22 + ... + (k + 1)2 =(k + 1)(k + 2)(2k + 3)

6

65

Page 66: Invariantes. Todo lo que debías saber para la OMA

Usando las identidades anteriores podemos probar otras más complicadas:

n∑k=1

(2k − 1)(n− k + 1) =n∑

k=1

2kn− 2k2 + 2k − n + k − 1

n∑k=1

(2k − 1)(n− k + 1) =n∑

k=1

(2n + 3)k − 2k2 − (n + 1)

n∑k=1

(2k − 1)(n− k + 1) = (2n + 3)n∑

k=1

k − 2n∑

k=1

k2 − n(n + 1)

n∑k=1

(2k − 1)(n− k + 1) = (2n + 3)n(n + 1)

2− 2n(n + 1)(2n + 1)

6− n(n + 1)

n∑k=1

(2k − 1)(n− k + 1) =3(2n + 3)n(n + 1)− 2n(n + 1)(2n + 1)− 6n(n + 1)

6

n∑k=1

(2k − 1)(n− k + 1) =n(n + 1)(3(2n + 3)− 2(2n + 1)− 6)

6

n∑k=1

(2k − 1)(n− k + 1) =n(n + 1)(2n + 1)

6

66

Page 67: Invariantes. Todo lo que debías saber para la OMA

3.3. Sucesiones recurrentesLa sucesión de Fibonacci comienza con 0 y 1 y cada término se obtiene como la suma

de los 2 anteriores.

0, 1, 1, 2, 3, 5, 8, 13, 21, 34, 55, 89, 144, 233, 377, 610

Si llamamos Fn al n−ésimo término comenzando por F0 = 0, F1 = 1 entonces {Fn}satisface:

Fn+2 = Fn+1 + Fn

Nuestro objetivo es dar una fórmula para Fn. Para ello primero hallemos otras sucesionesmás simples que satisfagan la misma recurrencia. Por ejemplo, busquemos sucesiones de laforma:

1, x, x2, x3, ......, xn, xn+1, xn+2, ....

La condición de que cada término sea la suma de los 2 anteriores se traduce en que paratodo n

xn+2 = xn+1 + xn

Luego x2 = x + 1. Resolviendo la cuadrática llegamos a 2 posibles valores, x1 = 1+√

52 y

x2 = 1−√

52 . Es decir que para todo n

(1 +

√5

2)n+2 = (

1 +√

52

)n+1 + (1 +

√5

2)n

(1−

√5

2)n+2 = (

1−√

52

)n+1 + (1−

√5

2)n

Es fácil ver ahora que si ponemos an = αxn1 +βxn

2 (recordar que x1 = 1+√

52 y x2 = 1−

√5

2 )entonces

an+2 = an+1 + an

¿Será posible encontrar α y β de forma que an = Fn para todo n? ¡Sí! Por empezar sia0 = F0 = 0 y a1 = F1 = 1 luego

α + β = a0 = 0

α1 +

√5

2+ β

1−√

52

= a1 = 1

Resolviendo el sistema de ecuaciones hallamos que α = 1√5

y β = − 1√5. Es decir que

an =1√5(1 +

√5

2)n − 1√

5(1−

√5

2)n

67

Page 68: Invariantes. Todo lo que debías saber para la OMA

Entonces an+2 = an+1 + an, a0 = 0 y a1 = 1. Pero como {an} y {Fn} comienzan igualy cumplen la misma recurrencia entonces deben ser iguales. Es decir que

Fn =1√5(1 +

√5

2)n − 1√

5(1−

√5

2)n

Hagamos otro ejemplo para ver cómo funciona esto. Definamos la sucesión {Gn} porG0 = 6 y G1 = 13 y

Gn+2 = 5Gn+1 − 6Gn

Tratemos de hallar una fórmula para Gn. Los primeros términos de la sucesión son

6, 13, 29, 67, 161, 403, 1049, 2827, 7841, 22243, ....

Las sucesiones de la forma {xn} que satisfacen la misma recurrencia son aquellas talesque para todo n

xn+2 = 5xn+1 − 6xn

O lo que es lo mismo

x2 = 5x− 6

Las soluciones de la ecuación anterior son x = 2 y x = 3 entonces queremos hallar α yβ tales que

Gn = α · 2n + β · 3n

Pero G0 = 6 y G1 = 13, esto implica que

α + β = 6

2α + 3β = 13

Resolviendo el sistema anterior llegamos a que β = 1 y α = 5. Es decir que

Gn = 5 · 2n + 3n

68

Page 69: Invariantes. Todo lo que debías saber para la OMA

3.4. PermutacionesUna permutación π = (π(1), π(2), ..., π(n)) de 1, 2, ..., n es un reordenamiento de los

números 1, 2, ..., n. Por ejemplo, si n = 7 las siguientes son permutaciones de 1234567:

6437215, 1234567, 7531246, 4361275, 7654321

Para calcular cuántas permutaciones de 1, ..., n hay, alcanza notar que tenemos n opcionespara el primer lugar, luego quedan n − 1 opciones para el segundo, luego n − 2 para eltercero,...,2 opciones para el anteúltimo y 1 opción para el último lugar. En definitiva, hayn(n−1)(n−2).... 3 2 1 = n! permutaciones de 1, 2, ..., n. A continuación listamos las 5! = 120permutaciones de 1, 2, 3, 4, 5.

12345 14235 21345 24135 31245 34125 41235 43125 51234 5312412354 14253 21354 24153 31254 34152 41253 43152 51243 5314212435 14325 21435 24315 31425 34215 41325 43215 51325 5321412453 14352 21453 24351 31452 34251 41352 43251 51352 5324112534 14523 21534 24513 31524 34512 41523 43512 51423 5341212543 14532 21543 24531 31542 34521 41532 43521 51432 5342113245 15234 23145 25134 32145 35124 42135 45123 52134 5412313254 15243 23154 25143 32154 35142 42153 45132 52143 5413213425 15324 23415 25314 32415 35214 42315 45213 52314 5421313452 15342 23451 25341 32451 35241 42351 45231 52341 5423113524 15423 23514 25413 32514 35412 42513 45312 52413 5431213542 15432 23541 25431 32541 35421 42531 45321 52431 54321

Cuadro 1: S5 = 120 permutaciones de 1,2,3,4,5

Si tenemos una permutación π = (π(1), π(2), ..., π(n)), una “inversión“ es un par i, j talque i < j y π(i) > π(j). La cantidad de inversiones de π se denota por α(π) y el signo de lapermutación es sg (π) = (−1)α(π). Por ejemplo

π = (35214) → α(π) = 6 → sg (π) = 1

π = (15342) → α(π) = 5 → sg (π) = −1

π = (14325) → α(π) = 3 → sg (π) = −1

El conjunto de permutaciones de 1, 2, ..., n se llama Sn. En realidad, Sn no es solamente unconjunto sino un “grupo“ pues se puede operar con las permutaciones de forma natural. Paraello tomemos n letras ordenadas y consideremos cada permutación π = (π(1), π(2), ..., π(n) )como una orden que reordena las n letras de forma que la primer letra quede en el lugar

69

Page 70: Invariantes. Todo lo que debías saber para la OMA

π(1), la segunda letra en el lugar π(2), la tercera en π(3),...., la última en π(n). Por ejemplosi n = 5.

(13425)(ABCDE) = ADBCE

(21345)(ABCDE) = BACED

(51234)(ABCDE) = BCDEA

Cuando aplicamos más de una orden, éstas se leen de derecha a izquierda, por ejemplo:

(12534)(43521)(54321)(ABCDE) = (12534)(43521)(EDCBA)

(12534)(43521)(54321)(ABCDE) = (12534)(ABDEC)

(12534)(43521)(54321)(ABCDE) = (ABECD)

Es fácil deducir “reglas para reemplazar“ 2 permutaciones por otra. Por ejemplo:

(12453)(21345)(ABCDE) = (12453)(BACED) = BADCE = (21435)(ABCDE)

(21345)(12453)(ABCDE) = (21345)(ABECD) = BAECD = (21453)(ABCDE)

Luego (12453)(21345) = (21435) y (21345)(12453) = (21453). La ventaja de pensar laspermutaciones como órdenes es que podemos operar con ellas; si π1, π2 son dos permutaciónesentonces π2π1 es la permutación que ordena las letras como lo hace primero aplicar π1 ydespués π2.

70

Page 71: Invariantes. Todo lo que debías saber para la OMA

3.5. MenelaoSupongamos que tenemos un triángulo abc y puntos x, y, z en ab, bc, ca respectivamente.

¿Cómo saber si xyz están alineados?

Teorema de Menelao: Sea abc un triángulo y L una recta que corta a los lados ab, bc, caen x, y, z respectivamente. Luego

ax

bx

by

cy

cz

az= 1

Demostración: Sean ha, hb, hc los pies de las perpendiculares desde a, b, c a la recta L.Luego los triángulos ahax y bhbx son semejantes de donde

ax

bx=

aha

bhb(1)

Análogamente

by

cy=

bhb

chc(2)

cz

az=

chc

aha(3)

71

Page 72: Invariantes. Todo lo que debías saber para la OMA

Multiplicando (1), (2) y (3):

ax

bx

by

cy

cz

az=

aha

bhb

bhb

chc

chc

aha

ax

bx

by

cy

cz

az= 1

Como queríamos. ‡

Ahora veamos el “Dual“ de Menelao, el Teorema de Ceva.

Teorema de Ceva: Sea abc un triángulo y x, y, z puntos en los lados ab, bc, ca respec-tivamente tales que cx, ay, bz se cortan en un punto p. Entonces

ax

bx

by

cy

cz

az= 1

Demostración: Recordemos que si tenemos 2 triángulos con la misma altura, la razónde sus áreas es la razón de sus bases. Luego

by

yc=

(aby)(ayc)

=(bpy)(cpy)

=(aby)− (bpy)(ayc)− (cpy)

=(abp)(cap)

by

yc=

(bap)(cap)

Análogamentecz

za=

(cpb)(apb)

ax

xb=

(apc)(bpc)

De donde

72

Page 73: Invariantes. Todo lo que debías saber para la OMA

ax

bx

by

cy

cz

az=

(apc)(bpc)

(bap)(cap)

(cpb)(apb)

ax

bx

by

cy

cz

az= 1

Como queríamos. ‡

Es fácil ver que valen los recíprocos de Menelao y Ceva aunque con un pequeño detalle,como no usamos segmentos dirigidos vamos a decirlo de la siguiente manera.

“Sea abc un triángulo y x, y, z puntos en los lados ab, bc, ca (o en sus prolonga-ciones) respectivamente. Supongamos que

ax

bx

by

cy

cz

az= 1

Entonces xyz están alineados o ay, bz, cx se cortan en un punto 16.“

Hagamos una observación más. Supongamos que tenemos un triángulo abc y puntosx, y, z puntos en los lados ab, bc, ca respectivamente tales que cx, ay, bz se cortan en unpunto p. Marquemos ahora y′ la intersección de la recta xz con bc. Luego por el Teoremade Ceva y el Teorema de Menelao:

ax

bx

by

cy

cz

az= 1

ax

bx

by′

cy′cz

az= 1

16Si hubiéramos usado segmentos dirigidos entonces se cumple que están alineados si el productoes −1 y son concurrentes si el producto es 1

73

Page 74: Invariantes. Todo lo que debías saber para la OMA

De donde

by

cy=

by′

cy′

Es decir que17 {bc : yy′} = 1. Tales puntos se dicen “conjugados harmónicos“ y es unapropiedad invariante por proyección (pues de hecho la razón doble lo es). Los conjugadosharmónicos tienen la siguiente propiedad:

“Sean a, b, c, d cuatro puntos distintos en una recta conjugados harmónicos, esdecir tales que {ac : bd} = 1. Tomemos p fuera de la recta y cualquier punto qen la recta bp. Luego sea r la intersección de cq con ap y s la intersección de aqcon cp, entonces r, s, d están alineados.“

17Para la definción de la razón doble ir a “Un poco de Geometría Proyectiva“.

74

Page 75: Invariantes. Todo lo que debías saber para la OMA

4. Problemas

75

Page 76: Invariantes. Todo lo que debías saber para la OMA

4.1. Algunos problemas para pensar1. Los números 1, 2, 3, ..., 30 están escritos en un círculo en un orden arbitrario. En cada

turno se puede intercambiar 2 números consecutivos. Si al final cada número ocupa laposición diametralmente opuesta a la inicial. Probar que en algún momento 2 númeroscon suma 31 fueron intercambiados.

2. Se dispone de un tablero de una sola fila pero infinitas casillas y de un número finito deporotos que se ubican ocupando casillas del tablero. Se realiza una sucesión de movidascomo sigue: en cada paso, se elige una casilla que tenga más de un poroto; se toman dosde esos porotos y se colocan uno en la casilla que está inmediatamente a la derechay el otro en la casilla que está inmediatamente a la izquierda de la casilla elegida.La sucesión de movidas finaliza si en algún momento hay a lo sumo un poroto encada casilla. Dada una configuración inicial, demostrar que toda sucesión de movidasfinalizará después de la misma cantidad de pasos y con la misma configuración.

3. (San Petersburgo 1997) El número 999..,999 (1997 nueves) está escrito en el pizarrón.Cada minuto uno de los números del pizarrón se factoriza como producto de 2 números,se borra el número y se reemplaza por sus 2 factores aumentados o disminuidos en2 (independientemente). ¿Es posible que en algún momento todos los números delpizarrón sean iguales a 9?

4. (Moscú 2002) Tenemos un collar con pn perlas (p primo y n ≥ 2). En cada operación elcollar se corta en varios pedazos del mismo largo, se da vuelta el orden de las perlas encada pedazo y se rearma el collar con los pedazos en el mismo orden en que estaban.¿Se puede obtener cualquier orden de las perlas mediante tales operaciones?

5. (Torneo de Las Ciudades 1980) Se tienen algunos puntos alrededor de un círculo cadauno de ellos coloreados de blanco o negro. Se pueden realizar 2 tipos de operaciones:

a) Se cambian de color 2 puntos consecutivos y se agrega un punto blanco entreambos.

b) Se quita un punto blanco y se cambia el color de los 2 vecinos.

Probar que si inicialmente sólo hay 2 puntos blancos no se puede conseguir tener sólo2 puntos negros.

6. (Torneo de Las Ciudades 1983) Se tiene un tablero de ajedrez infinito y en cada casillahay un rey salvo por un conjunto de casillas C. Una movida consiste en mover algunosreyes (posiblemente todos) a una casilla vecina (cada casilla tiene 8 vecinas) de formaque en cada casilla quede a lo sumo 1 rey.

a) Si C es el conjunto de casillas (m,n) con m y n múltiplos de 100. ¿Es posibleque luego de una cantidad finita de movidas cada casilla tenga un rey?

76

Page 77: Invariantes. Todo lo que debías saber para la OMA

b) Si C es el conjunto de casillas ocupadas o atacadas por un conjunto arbitrariode damas en el tablero. ¿Es posible que luego de una cantidad finita de movidascada casilla tenga un rey?

7. (Torneo de Las Ciudades 1987) Se tiene un triángulo equilátero de lado n divididoen n2 triángulos equiláteros de lado 1. Inicialmente hay un triángulo de blanco y elresto de negro. En cada turno se puede cambiar el color de todos los triángulos entre2 paralelas consecutivas a un lado del triángulo. ¿Puede conseguirse que todos lostriángulos sean negros?

8. (Torneo de Las Ciudades 1993) Hay 4 ranas. En cada turno una rana salta. Si la ranaque salta estaba en A y las otras en B,C y D entonces la rana salta hasta E tal queel punto medio de AE sea el baricentro de BCD. Si inicialmente las ranas estaban enlos vértices de un cuadrado, ¿es posible que luego de algunos turnos una rana salteencima de otra?

9. (Torneo de Las Ciudades 2005) Inicialmente no hay ningún número en el pizarrón,en cada etapa se puede o bien escribir dos 1’s o bien borrar dos números idénticos,digamos n y al mismo tiempo escribir los dos números n + 1 y n − 1. Determinarel número mínimo de etapas necesarias para que al menos uno de los números delpizarrón sea el 2005.

10. (Leningrado 1988) Se tiene una pila con 1001 piedras. En cada turno se elige una pila,se le quita una piedra y el resto se divide en 2 pilas nuevas. ¿Es posible lograr quetodas las pilas tengan 3 piedras?

11. (Leningrado 1988) Cualquier secuencia de 0′s y 1′s es una palabra. Un triplicado estomar una palabra y repetirla 3 veces (una a continuación de la otra). Por ejem-plo 010101, 111, 100110011001. Dada una palabra se la puede transformar en otraintroduciéndole o borrándole un triplicado en cualquier lugar. ¿Es posible cambiar lapalabra 01 en 10 por una secuencia de transformaciones como las recién descripta?

12. (Leningrado 1991) Se tienen varios números enteros escritos en un círculo. En cadaturno se puede quitar un número par y sumar los 2 números que eran vecinos a éste(.., x, 2y, z, ... → ..., x + y, ...). La operación se repite hasta que queden 1 o 2 númeroso que todos los números sean impares. Probar que la cantidad de números que quedenes independiente de lo que hagamos.

13. (Bamo 2006) Se tienen k manijas alineadas, cada una de los cuales está hacia arriba,abajo, derecha o izquierda. Cuando cualesquiera 3 manijas consecutivas apunten endirecciones distintas se pueden girar las 3 de forma que apunten a la cuarta dirección.Probar que esta operación no se puede repetir indefinidamente.

14. (Estonia 2002) Sea n un entero positivo dado. Pongamos una ficha en cada númeronegativo. En cada turno podemos elegir n enteros consecutivos, quitar una ficha de

77

Page 78: Invariantes. Todo lo que debías saber para la OMA

uno ellos y reacomodar las demás fichas de estos n enteros como queramos en los nenteros (a lo sumo una ficha por número). Decidir si existe n de forma que para cadaN > 0 es posible poner una ficha a la derecha de N .

15. Nos es dado un tablero de n × n con números enteros en sus casillas. En cada pasopodemos realizar una de las siguientes operaciones:

-Sumar a una fila un múltiplo de otra

-Sumar a una columna un múltiplo de otra

-Intercambiar 2 filas

-Intercambiar 2 columnas

Probar que se puede llevar al tablero a uno con los números (d1, ..., dn) en su diagonaly 0 fuera de ella de forma que d1|d2, d2|d3,... ,dn−1|dn. Probar además que los númerosd1, ..., dn son únicos salvo por su signo (esta es la forma normal de Smith).

16. (Tercer problema de Hilbert) ¿Es posible cortar un cubo en una cantidad finita depoliedros y con ellos armar un tetraedro?

78